SlideShare uma empresa Scribd logo
1 de 169
EXAMINATION OFEXAMINATION OF
THE ABDOMENTHE ABDOMEN
Dr.Sreejoy PatnaikDr.Sreejoy Patnaik
ACUTE ABDOMENACUTE ABDOMEN
One of the most common causes for hospitalizationOne of the most common causes for hospitalization
Meaning = acute abdominal symptoms which leadMeaning = acute abdominal symptoms which lead
patients to ER , excluding obvious abdominalpatients to ER , excluding obvious abdominal
injuriesinjuries
May or may not require immediate operationsMay or may not require immediate operations
Acute Abdomen ObjectivesAcute Abdomen Objectives
Definition of acute abdomen.Definition of acute abdomen.
To be able to distinguish between a medical orTo be able to distinguish between a medical or
surgical abdomensurgical abdomen
To be able to obtain a history to facilitate theTo be able to obtain a history to facilitate the
diagnosis:diagnosis:
Immediate Management of Life threateningImmediate Management of Life threatening
problems: perform a brief examination, identifyproblems: perform a brief examination, identify
candidates for urgent surgery.candidates for urgent surgery.
Further evaluation of patient with acute abdominalFurther evaluation of patient with acute abdominal
pain: H & P / lab. INV./ X-Rays/ special studies.pain: H & P / lab. INV./ X-Rays/ special studies.
Acute Abdomen objectivesAcute Abdomen objectives
Identify and describe the common localized abdominalIdentify and describe the common localized abdominal
massesmasses
- umbilical hernia, incisional hernia, epigastric hernia,- umbilical hernia, incisional hernia, epigastric hernia,
diastases recti, and lipoma.diastases recti, and lipoma.
Describe normal and abnormal-Describe normal and abnormal-
- Bowel sounds ( exaggerated)- Bowel sounds ( exaggerated)
- Bruits, Venous hum, Friction rubs- Bruits, Venous hum, Friction rubs
Acute AbdomenAcute Abdomen
objectivesobjectives
Identify the areas commonly auscultated forIdentify the areas commonly auscultated for
BRUITS. i.e., renal artery stenosisBRUITS. i.e., renal artery stenosis
Describe the significance of the abdomen in ALLDescribe the significance of the abdomen in ALL
FOUR QUADRANTSFOUR QUADRANTS
Describe the common abnormalities that can causeDescribe the common abnormalities that can cause
IRREGULAR PERCUSSION NOTES of theIRREGULAR PERCUSSION NOTES of the
abdomenabdomen
-ovarian tumor, pregnant uterus or GI obstruction-ovarian tumor, pregnant uterus or GI obstruction
Acute Abdomen objectivesAcute Abdomen objectives
Define and describe the changes associated with lightDefine and describe the changes associated with light
and deep palpationand deep palpation
To assess any degree of tenderness, i.e. reboundTo assess any degree of tenderness, i.e. rebound
tenderness, guardingtenderness, guarding
=peritoneal irritation or distended viscous.=peritoneal irritation or distended viscous.
Acute abdomenAcute abdomen
objectivesobjectives
Recognize and perform the special techniques inRecognize and perform the special techniques in
examining the abdomen for specific findings:examining the abdomen for specific findings:
(1) rebound tenderness(1) rebound tenderness
(2) shifting dullness in ascites(2) shifting dullness in ascites
(3) Fluid wave in ascites(3) Fluid wave in ascites
(4) hooking technique for palpating the liver(4) hooking technique for palpating the liver
(5) Murphy's sign for acute cholecystitis(5) Murphy's sign for acute cholecystitis
(6) ballottement(6) ballottement
(7) the techniques of assessing possible appendicitis(7) the techniques of assessing possible appendicitis
Acute AbdomenAcute Abdomen
Differential Dx. of Acute Abdominal Pain -Differential Dx. of Acute Abdominal Pain -
Anatomic correlation of abdominal painAnatomic correlation of abdominal pain
Types of abdominal painTypes of abdominal pain
Causes of abdominal pain by quadrantsCauses of abdominal pain by quadrants
Parietal pain, visceral pain and referred pain.Parietal pain, visceral pain and referred pain.
What does the pain feel likeWhat does the pain feel like ??
Sharp Pain : Biliary ColicSharp Pain : Biliary Colic
Tearing Pain : Aortic DissectionTearing Pain : Aortic Dissection
PainPain
Penetrating Pain : PancreatitisPenetrating Pain : Pancreatitis
Acute AbdomenAcute Abdomen
H&P: Obtain a complete HxH&P: Obtain a complete Hx
Vital signs –Vital signs –
Blood pressure ( standing or sitting position)Blood pressure ( standing or sitting position)
Pulse & asses peripheral perfusionPulse & asses peripheral perfusion
AlertnessAlertness
Temperature of skin and extremitiesTemperature of skin and extremities
Immediate management of life threateningImmediate management of life threatening
Problems: bleeding, shock, hypotensionProblems: bleeding, shock, hypotension
Acute AbdomenAcute Abdomen
Location of Problem: Chest, abdomen (upper,Location of Problem: Chest, abdomen (upper,
middle, lower/ sides)middle, lower/ sides)
Time of Onset: Date, time…?Time of Onset: Date, time…?
Type of Onset: How: Sudden? Gradual?Type of Onset: How: Sudden? Gradual?
Original Source: Triggers, what were youOriginal Source: Triggers, what were you
doing? (setting at time of occurrence)doing? (setting at time of occurrence)
Severity: Interfere with ADL’S? (Activity ofSeverity: Interfere with ADL’S? (Activity of
daily living)daily living)
Time Relationship: How often, when?Time Relationship: How often, when?
Duration: How long an episode?Duration: How long an episode?
Acute AbdomenAcute Abdomen
Course: Getting better, worse?Course: Getting better, worse?
Association: Any other manifestation?Association: Any other manifestation?
Source of Relief: Changes inSource of Relief: Changes in
medication, diet? What makes itmedication, diet? What makes it
better?better?
Source of Aggravation: What makes itSource of Aggravation: What makes it
worse?worse?
Relevant Data & Pertinent NegativesRelevant Data & Pertinent Negatives
Acute AbdomenAcute Abdomen
Gynecological Hx for females: last menstrualGynecological Hx for females: last menstrual
period, pregnancies, STD’s.period, pregnancies, STD’s.
Associated symptoms: nausea, anorexia,Associated symptoms: nausea, anorexia,
vomiting, change in bowel habits.vomiting, change in bowel habits.
Acute AbdomenAcute Abdomen
Dyspnea, SOB, pain, wheezing, crackles, orthopnea,Dyspnea, SOB, pain, wheezing, crackles, orthopnea,
(?) Pillows, cough, sputum, emphysema, bronchitis,(?) Pillows, cough, sputum, emphysema, bronchitis,
asthma, URL, chest x-rayasthma, URL, chest x-ray
Changes in:Changes in:
appetite, weight, N/V. abdominal pain, (?) Diet,appetite, weight, N/V. abdominal pain, (?) Diet,
digestion, tastes, bowel habits/ stool.digestion, tastes, bowel habits/ stool.
Urine: color, polyuria, oliguria, nocturia, dysuria,Urine: color, polyuria, oliguria, nocturia, dysuria,
frequency, urgency, stones…frequency, urgency, stones…
Acute AbdomenAcute Abdomen
Mental statusMental status
Abdominal examination (gently) looking for signs ofAbdominal examination (gently) looking for signs of
acute abdomen.acute abdomen.
Identify and describe the common localizedIdentify and describe the common localized
abdominal massesabdominal masses
Pelvic(gynecological) exam for females and rectalPelvic(gynecological) exam for females and rectal
exam for both male and femaleexam for both male and female
(gross blood, asses sphincter tone, and any other(gross blood, asses sphincter tone, and any other
evidence of trauma).evidence of trauma).
Check for blood in stools: UC, diverticular ds orCheck for blood in stools: UC, diverticular ds or
diverticulitis, haemorroidsdiverticulitis, haemorroids
PARTS OF THE GUTPARTS OF THE GUT
Stomach to 2ndpart of duodenum, includingStomach to 2ndpart of duodenum, including
liver, biliary trees, pancreas, and spleen areliver, biliary trees, pancreas, and spleen are
derived from FOREGUTderived from FOREGUT
33rdrd
and 4thpart of duodenum, jejunum, ileum,and 4thpart of duodenum, jejunum, ileum,
appendix, ascending colon to proximal 2/3 ofappendix, ascending colon to proximal 2/3 of
transverse colon are derived from MIDGUTtransverse colon are derived from MIDGUT
Distal 1/3 of transverse colon to anal canalDistal 1/3 of transverse colon to anal canal
above dentate line are derived fromabove dentate line are derived from
HINDGUTHINDGUT
Pathophysiology of Abdominal painPathophysiology of Abdominal pain
Visceral pain from organs derived from FOREGUTVisceral pain from organs derived from FOREGUT
and MIDGUT is at midline and above or around theand MIDGUT is at midline and above or around the
umbilicus.umbilicus.
Visceral pain from organs derived from HINDGUT isVisceral pain from organs derived from HINDGUT is
at midline and below the umbilicusat midline and below the umbilicus..
ANATOMIC ESSENTIALSANATOMIC ESSENTIALS
Abdominal pain isAbdominal pain is
typically derived fromtypically derived from
one or more threeone or more three
distinct pain pathwaysdistinct pain pathways
1. Visceral,1. Visceral,
2.Parietal (Somatic )2.Parietal (Somatic )
3.Referred3.Referred
Peritoneum innervationsPeritoneum innervations
VISCERAL PATHWAYVISCERAL PATHWAY
Sympathetic and parasympathetic nerveSympathetic and parasympathetic nerve
innervations(C fibers)innervations(C fibers)
CharacterCharacter
-dull or cramping pain-dull or cramping pain
-insidious-insidious
-sensitive to distension, ischemia, squeezing,-sensitive to distension, ischemia, squeezing,
torsiontorsion
-insensitive to heat, cutting, or electrical shock-insensitive to heat, cutting, or electrical shock
The visceral Abdominal painThe visceral Abdominal pain
Visceral pain fibers areVisceral pain fibers are
bilateral, unmyelinated andbilateral, unmyelinated and
enter the spinal cord atenter the spinal cord at
multiple levels .multiple levels .
The visceral abdominal pain isThe visceral abdominal pain is
usually dull , , poorly localizedusually dull , , poorly localized
and experienced in the midline .and experienced in the midline .
Visceral Abdominal PainVisceral Abdominal Pain
Visceral Abdominal PainVisceral Abdominal Pain
is usually caused byis usually caused by
distention of hollowdistention of hollow
organs or capsularorgans or capsular
stretching of solidstretching of solid
organs.organs.
Visceral Abdominal PainVisceral Abdominal Pain
Less commonly , it is caused by ischemia orLess commonly , it is caused by ischemia or
inflammation.inflammation.
The tissue congestion sensitizes nerve endings ofThe tissue congestion sensitizes nerve endings of
visceral pain fibers and lowers the threshold forvisceral pain fibers and lowers the threshold for
stimulus.stimulus.
Visceral Abdominal PainVisceral Abdominal Pain
If the involved organ isIf the involved organ is
affected by peristalsis,affected by peristalsis,
the pain is oftenthe pain is often
described asdescribed as
intermittent, cramp, orintermittent, cramp, or
colicky in naturecolicky in nature
PARIETAL PATHWAYPARIETAL PATHWAY
Somatic nerve innervations (A fibers)Somatic nerve innervations (A fibers)
Somatic nerve distribution (T7-L2, umbilicus at T12)Somatic nerve distribution (T7-L2, umbilicus at T12)
CharacterCharacter
-----sharp and exquisite pain-----sharp and exquisite pain
Sensitive to mechanical stimuli (stretching, pinprick,Sensitive to mechanical stimuli (stretching, pinprick,
pinch), heat, electrical shock, chemical stimulus,pinch), heat, electrical shock, chemical stimulus,
infection-inflammationinfection-inflammation
Parietal (Somatic) AbdominalParietal (Somatic) Abdominal
painpain
Results fromResults from
ischemia, ,ischemia, ,
inflammation , orinflammation , or
stretching of thestretching of the
parietalparietal
Peritoneum.Peritoneum.
Parietal (Somatic ) AbdominalParietal (Somatic ) Abdominal
PainPain
Myelinated afferent fibersMyelinated afferent fibers
that transmit the painfulthat transmit the painful
stimulus to specific dorsalstimulus to specific dorsal
root ganglia on the same sideroot ganglia on the same side
and dermatomal level at theand dermatomal level at the
origin of the painorigin of the pain
Parietal (Somatic ) AbdominalParietal (Somatic ) Abdominal
PainPain
The parietal pain , in contrast to visceral painThe parietal pain , in contrast to visceral pain
pain, , often can be localized to the region ofpain, , often can be localized to the region of
the painful stimulus.the painful stimulus.
• This pain is typically sharp, knife- like andThis pain is typically sharp, knife- like and
constant; coughing and moving are likelyconstant; coughing and moving are likely
likely to aggravate it.likely to aggravate it.
• The classic presentation ofThe classic presentation of
appendicitis involves bothappendicitis involves both
visceral and parietal pain.visceral and parietal pain.
The pain of early presentation isThe pain of early presentation is
often periumbilical (visceral )often periumbilical (visceral )
but localizes to the right lowerbut localizes to the right lower
quadrant ( RLQ) when thequadrant ( RLQ) when the
inflammation extends to theinflammation extends to the
peritoneum (parietalperitoneum (parietal).).
REFERRED PAINREFERRED PAIN
• Is defined as pain felt at aIs defined as pain felt at a
distance from the diseaseddistance from the diseased
organ .organ .
• It results from sharedIt results from shared
central pathways forcentral pathways for
afferent neurons fromafferent neurons from
different locations .different locations .
ASSESSMENTASSESSMENT
2 MOST IMPORTANT THINGS in assessment of the2 MOST IMPORTANT THINGS in assessment of the
patients are--patients are--
• Careful and precise history taking, andCareful and precise history taking, and
• Physical examinationPhysical examination
BASIC HISTORY TAKINGBASIC HISTORY TAKING
• ONSETONSET
• 1. SUDDEN1. SUDDEN
• - Perforated PU, Gallstone, UC, Aortic dissection,- Perforated PU, Gallstone, UC, Aortic dissection,
Rupture AAA, SMA Embolism, Ruptured ectopicRupture AAA, SMA Embolism, Ruptured ectopic
preg., Ruptured corpus lateral or follicular cysts,preg., Ruptured corpus lateral or follicular cysts,
Twisted ovarian cystTwisted ovarian cyst
• 2. INSIDIOUS2. INSIDIOUS
• - Acute Appendicitis, Acute pancreatitis, Intestinal- Acute Appendicitis, Acute pancreatitis, Intestinal
obstruction, Acute pyelonephritis, Acute gastritis orobstruction, Acute pyelonephritis, Acute gastritis or
gastroenteritisgastroenteritis
AGEAGE
1.1. CHILDHOOD:CHILDHOOD:
Constipation, Acute appendicitis, Intussusceptions,Constipation, Acute appendicitis, Intussusceptions,
Viral enteritisViral enteritis
2.ADULTS::2.ADULTS::
Infecion-inflammation,Female reproductive organsInfecion-inflammation,Female reproductive organs
3.MIDDLE TO OLD AGE :3.MIDDLE TO OLD AGE :
Malignancies, Degenerative diseasesMalignancies, Degenerative diseases
NATURE OF PAINNATURE OF PAIN
COLICKY SHARPSHOOTING,COLICKY SHARPSHOOTING,
intermittent, restless, associated withintermittent, restless, associated with
vomitingvomiting
-is likely from acute obstruction of hollow-is likely from acute obstruction of hollow
viscous organs (small bowel, biliary trees,viscous organs (small bowel, biliary trees,
ureter,or even appendix).ureter,or even appendix).
-SMA occlusion maybe possible-SMA occlusion maybe possible
-Acute Gastritis, Gastroenteritis-Acute Gastritis, Gastroenteritis
SUDDEN,SHARP & PERSISTENT:SUDDEN,SHARP & PERSISTENT:
-Leakage of irritating fluid, i.e. blood from-Leakage of irritating fluid, i.e. blood from
Ruptured ectopic preg,Ruptured ectopic preg,
-AAA,-AAA,
-Corpus luteal or follicular cysts, --Corpus luteal or follicular cysts, -
-HaematomaFluid from ovarian cyst,-HaematomaFluid from ovarian cyst,
-Perforated PU-Perforated PU
SHEARING OR TEARING:SHEARING OR TEARING:
-Aortic dissection,-Aortic dissection,
-Ruptured AAA-Ruptured AAA
• ASSOCIATED SYMPTOMSASSOCIATED SYMPTOMS : Nausea,vomiting, resp.: Nausea,vomiting, resp.
symptomssymptoms
• BOWEL HABITS :BOWEL HABITS : Diarrhea,constipation,Diarrhea,constipation,
mucous bloody stoolmucous bloody stool
• GYNECOLOGIC HISTORYGYNECOLOGIC HISTORY ::
menstruation,leucorrhea, sexual intercoursemenstruation,leucorrhea, sexual intercourse
• CONCOMITANT HISTORYCONCOMITANT HISTORY::
• Underlying diseasesUnderlying diseases
• Family HistoryFamily History
• Drug usageDrug usage
• Substance expSubstance exposureosure
ABDOMINALASSESSMENTABDOMINALASSESSMENT
LANDMARKSLANDMARKS
• 1. Xiphoid Process1. Xiphoid Process
• 2. Costal Margin2. Costal Margin
• 3. Abdominal Midline3. Abdominal Midline
• 4. Umbilicus4. Umbilicus
• 5. Rectus Abdominis Muscle5. Rectus Abdominis Muscle
• 6.Ant. Sup. Iliac Spine6.Ant. Sup. Iliac Spine
• 7. Inguinal Ligament7. Inguinal Ligament
(Poupart’s Ligament)(Poupart’s Ligament)
• 8. Symphysis Pubis8. Symphysis Pubis
Position of the patient and exposure forPosition of the patient and exposure for
abdominal examination. Note that theabdominal examination. Note that the
genitalia must be exposed.genitalia must be exposed.
ASSESSMENT OF THE ABDOMENASSESSMENT OF THE ABDOMEN
INSPECTION -- CONTOURINSPECTION -- CONTOUR
• TYPES OFTYPES OF
ABDOMEN:ABDOMEN:
• FlatFlat
• rounded or convexrounded or convex
• ScaphoidScaphoid
• protuberantprotuberant
Assessment of the AbdomenAssessment of the Abdomen
Inspection -- ContourInspection -- Contour
• Flat is normalFlat is normal
LARGE CONVEX ABDOMEN -- 7 F’SLARGE CONVEX ABDOMEN -- 7 F’S
FatFat
FaecesFaeces
Fluid (ascites)Fluid (ascites)
FoetusFoetus
FlatusFlatus
Fatal growth (malignancy)Fatal growth (malignancy)
Fibroid tumorFibroid tumor
ASSESSMENT OF THE ABDOMENASSESSMENT OF THE ABDOMEN
INSPECTION -- CONTOURINSPECTION -- CONTOUR
• CONCAVE OR SCAPHOID ABDOMENCONCAVE OR SCAPHOID ABDOMEN
– Decreased fat depositsDecreased fat deposits
– Malnourished stateMalnourished state
– Flaccid muscle toneFlaccid muscle tone
• CONVEX OR PROTUBERANT ABDOMEN -- 7CONVEX OR PROTUBERANT ABDOMEN -- 7
F’s: fat, fluid (ascites), flatus, faeces, foetus, fatalF’s: fat, fluid (ascites), flatus, faeces, foetus, fatal
growth (malignancy), fibroid tumorgrowth (malignancy), fibroid tumor
ASSESSMENT OF THE ABDOMENASSESSMENT OF THE ABDOMEN
INSPECTION -- SYMMETRYINSPECTION -- SYMMETRY
• THE ABDOMEN SHOULD BE SYMMETRICALTHE ABDOMEN SHOULD BE SYMMETRICAL
BILATERALBILATERAL
• Asymmetry indicatesAsymmetry indicates
– tumortumor
– cystscysts
– bowel obstructionbowel obstruction
– enlargement of abdominal organsenlargement of abdominal organs
– scoliosisscoliosis
ASSESSMENT OF THE ABDOMENASSESSMENT OF THE ABDOMEN
• INSPECTION -- RECTUS ABDOMINIS MUSCLEINSPECTION -- RECTUS ABDOMINIS MUSCLE
• Normal -- no ridge separating the musclesNormal -- no ridge separating the muscles
• When a ridge is present - diastasis recti abdominusWhen a ridge is present - diastasis recti abdominus
– marked obesitymarked obesity
– past pregnancypast pregnancy
– increased intra-abdominal pressureincreased intra-abdominal pressure
ASSESSMENT OF THE ABDOMENASSESSMENT OF THE ABDOMEN
RESPIRATORY MOVEMENTRESPIRATORY MOVEMENT
INSPECTIONINSPECTION
• Normally no retractions -- the abdomen rises andNormally no retractions -- the abdomen rises and
falls with each respirationfalls with each respiration
• Abnormal due to abdominal disordersAbnormal due to abdominal disorders
– appendicitis with local peritonitisappendicitis with local peritonitis
– pancreatitispancreatitis
– biliary colic or ac. cholecystitisbiliary colic or ac. cholecystitis
– perforated ulcerperforated ulcer
ASSESSMENT OF THE ABDOMENASSESSMENT OF THE ABDOMEN
• Masses or nodulesMasses or nodules
-- tumors, metastasis, internal malignancy, or-- tumors, metastasis, internal malignancy, or
pregnancypregnancy
• Visible PeristalsisVisible Peristalsis
-- indicative of obstruction-- indicative of obstruction
• PulsationPulsation
-- aortic aneurysm or may occur in aortic-- aortic aneurysm or may occur in aortic
regurgitation and in right ventricular hypertrophyregurgitation and in right ventricular hypertrophy
ASSESSMENT OF THE ABDOMENASSESSMENT OF THE ABDOMEN
UMBILICUSUMBILICUS
• Umbilical HerniaUmbilical Hernia -- protrusion of the umbilicus-- protrusion of the umbilicus
secondary to non closure of the ring permittingsecondary to non closure of the ring permitting
intestine or omentum to protrudeintestine or omentum to protrude
• Sister Mary Joseph’s noduleSister Mary Joseph’s nodule -- nodule in the-- nodule in the
umbilicus secondary to CA intra-abdominalumbilicus secondary to CA intra-abdominal
• Intra-abdominal pressureIntra-abdominal pressure -- protrusion from ascites,-- protrusion from ascites,
masses, or pregnancymasses, or pregnancy
Some commonly used abdominal incisions. The midlineSome commonly used abdominal incisions. The midline
and oblique incisions avoid damage to the innervation ofand oblique incisions avoid damage to the innervation of
the abdominal musculature and the later developmentthe abdominal musculature and the later development
of incisional herniaeof incisional herniae
Assessment of the AbdomenAssessment of the Abdomen
PALPATIONPALPATION
• Please warm your handsPlease warm your hands
• To start avoid known tender areasTo start avoid known tender areas
• Tell the patient take slow deep breaths throughTell the patient take slow deep breaths through
mouthmouth
• Begin with gentle pressure then increaseBegin with gentle pressure then increase
• If ticklish or with children palpate throughIf ticklish or with children palpate through
their hand till ticklishness is gonetheir hand till ticklishness is gone
• Avoid quick, short jabsAvoid quick, short jabs
• Observe patient’s face for expressions of painObserve patient’s face for expressions of pain
Downloaded from: StudentConsult (on 25 March 2012 05:22 PM)
© 2005 Elsevier
Correct method of palpation. The hand is held flat
and relaxed, and 'moulded' to the abdominal wall.
Downloaded from: StudentConsult (on 25 March 2012 05:22 PM)
© 2005 Elsevier
Incorrect method of palpation. The hand is held
rigid and mostly not in contact with the
abdominal wall.
Method of deep palpation in an obese, muscular or
poorly relaxed patient.
Assessment of the AbdomenAssessment of the Abdomen
PALPATION -- LIGHTPALPATION -- LIGHT
• Lightly palpate to noteLightly palpate to note
– Skin temperatureSkin temperature
– TendernessTenderness
– Large massesLarge masses
Assessment of the AbdomenAssessment of the Abdomen
Palpation -- Abdominal Muscle GuardingPalpation -- Abdominal Muscle Guarding
• Use both hands -- one on each rectusUse both hands -- one on each rectus
• Check for tensing during expirationCheck for tensing during expiration
• When positive it is indicative of peritonealWhen positive it is indicative of peritoneal
irritation -- peritonitisirritation -- peritonitis
Assessment of the AbdomenAssessment of the Abdomen
Palpation -- DeepPalpation -- Deep
• You can use one or two handed methodYou can use one or two handed method
• Two handed method is usually used in obese orTwo handed method is usually used in obese or
very muscular individualsvery muscular individuals
• Palpate all quadrantsPalpate all quadrants
Assessment of the AbdomenAssessment of the Abdomen
Palpation -- Fluid WavePalpation -- Fluid Wave
• With an assistantWith an assistant
placing the ulnarplacing the ulnar
surface of their handsurface of their hand
firmly in the midlinefirmly in the midline
of the patientof the patient
• You tap from one sideYou tap from one side
to feel the wave on theto feel the wave on the
other sideother side
• Present withPresent with ascitesascites
Assessment of the AbdomenAssessment of the Abdomen
PalpationPalpation ---- Liver -- Bimanual MethodLiver -- Bimanual Method
• Left hand under patient’sLeft hand under patient’s
right flank (11th-12thright flank (11th-12th
rib) press upwardrib) press upward
• Place right hand at levelPlace right hand at level
of dullness -- have patientof dullness -- have patient
take a deep breathtake a deep breath
• Push in deeplyPush in deeply
• note -- size, shape,note -- size, shape,
consistency, or massesconsistency, or masses
Assessment of the AbdomenAssessment of the Abdomen
Palpation -- Liver -- Hook MethodPalpation -- Liver -- Hook Method
• Place both hands side byPlace both hands side by
side below the level of liverside below the level of liver
dullnessdullness
• Hook fingers in and up andHook fingers in and up and
have the patient take a deephave the patient take a deep
breath inbreath in
• Note the size, shape,Note the size, shape,
consistency, and any massesconsistency, and any masses
• I prefer to do this in a sittingI prefer to do this in a sitting
positionposition
Assessment of the AbdomenAssessment of the Abdomen
Palpation -- Liver -- HepatomegalyPalpation -- Liver -- Hepatomegaly
Enlarged liverEnlarged liver
– congestive heart failurecongestive heart failure
– hepatitishepatitis
– encephalopathyencephalopathy
– cirrhosiscirrhosis
– cystcyst
– cancercancer
Assessment of the AbdomenAssessment of the Abdomen
Palpation -- Liver -- Murphy’s SignPalpation -- Liver -- Murphy’s Sign
• Palpate the liver margin at the lateral borderPalpate the liver margin at the lateral border
of the rectus muscleof the rectus muscle
• Have the patient take a deep breathHave the patient take a deep breath
• If patient exhibits pain and stops inhaling thisIf patient exhibits pain and stops inhaling this
is a positiveis a positive Murphy’s SignMurphy’s Sign present inpresent in
CholecystitisCholecystitis
Assessment of the AbdomenAssessment of the Abdomen
Palpation -- Spleen -- Bimanual TechniquePalpation -- Spleen -- Bimanual Technique
• Pull up with left hand andPull up with left hand and
push in with right hand onpush in with right hand on
inspirationinspiration
• Will only be able to feel if 3Will only be able to feel if 3
times normal sizetimes normal size
• SplenomegalySplenomegaly
– inflammationinflammation
– congestive heart failurecongestive heart failure
– cancercancer
– cirrhosiscirrhosis
Assessment of the AbdomenAssessment of the Abdomen
Palpation -- Kidney -- Bimanual TechniquePalpation -- Kidney -- Bimanual Technique
• Place one hand on thePlace one hand on the
costovertebral angle of thecostovertebral angle of the
back and the other handback and the other hand
just below the costaljust below the costal
marginmargin
• Increase pressure duringIncrease pressure during
inspiration then haveinspiration then have
patient hold breathpatient hold breath
Assessment of the AbdomenAssessment of the Abdomen
Palpation -- KidneyPalpation -- Kidney
• The right kidney maybe difficult to distinguishThe right kidney maybe difficult to distinguish
from an enlarged liverfrom an enlarged liver
• Left kidney enlargement maybe difficult toLeft kidney enlargement maybe difficult to
distinguish from an enlarged spleendistinguish from an enlarged spleen
• Enlarged palpable kidneys are secondary to:Enlarged palpable kidneys are secondary to:
– hydronephrosishydronephrosis
– neoplasmsneoplasms
– polycystic diseasepolycystic disease
Assessment of the AbdomenAssessment of the Abdomen
Palpation -- AortaPalpation -- Aorta
• Assess the width of theAssess the width of the
aorta by placing youraorta by placing your
hands on each side ofhands on each side of
the aorta just abovethe aorta just above
the umbilicusthe umbilicus
• Abdominal aorticAbdominal aortic
aneurysmaneurysm -- width-- width
greater than 4 cm withgreater than 4 cm with
lateral pulsationslateral pulsations
AssesAssessment of the Abdomensment of the Abdomen
Rebound TendernessRebound Tenderness
• Apply firm pressure for several seconds to theApply firm pressure for several seconds to the
abdomen with hand at right angles and fingersabdomen with hand at right angles and fingers
extendedextended
• Quickly release the pressureQuickly release the pressure
• Test away from site where pain is initially determinedTest away from site where pain is initially determined
Assessment of the AbdomenAssessment of the Abdomen
Rebound TendernessRebound Tenderness
• Pain at site is direct rebound tendernessPain at site is direct rebound tenderness
• Pain at another site is referred reboundPain at another site is referred rebound
tendernesstenderness
• Indicative of peritoneal inflammationIndicative of peritoneal inflammation
• If in the RLQ think appendicitisIf in the RLQ think appendicitis
(McBurney’s point -- to be discussed later)(McBurney’s point -- to be discussed later)
Assessment of the AbdomenAssessment of the Abdomen
Rovsing’s SignRovsing’s Sign
• Press in the LLQ evenly for 5 secondsPress in the LLQ evenly for 5 seconds
and note if patient has pain in RLQ -and note if patient has pain in RLQ -
positivepositive
• Gas is pushed through the ileocecalGas is pushed through the ileocecal
valve thus distending the cecumvalve thus distending the cecum
• InIn appendicitisappendicitis pain is notedpain is noted
Assessment of the AbdomenAssessment of the Abdomen
Cutaneous HypersensitivityCutaneous Hypersensitivity
• Either lift the skin or stimulate the skin withEither lift the skin or stimulate the skin with
gentle jabbing with a sterile pingentle jabbing with a sterile pin
• Indicates a zone of peritoneal irritationIndicates a zone of peritoneal irritation
– RLQRLQ -- appendicitis-- appendicitis
– Mid Epigastrium --Mid Epigastrium -- peptic ulcerpeptic ulcer
Assessment of the AbdomenAssessment of the Abdomen
Iliopsoas Muscle TestIliopsoas Muscle Test
• Place your hand overPlace your hand over
the right thigh andthe right thigh and
push downward as thepush downward as the
patient is trying topatient is trying to
raise the leg, flexingraise the leg, flexing
the hipthe hip
• Positive RLQ painPositive RLQ pain
associated with aassociated with a
retrocecal orretrocecal or
perforated appendicitisperforated appendicitis
Assessment of the AbdomenAssessment of the Abdomen
Obturator Muscle TestObturator Muscle Test
• Flex the right leg at theFlex the right leg at the
hip and knee at a righthip and knee at a right
angle then rotate the legangle then rotate the leg
internally andinternally and
externallyexternally
• Pain indicative ofPain indicative of
inflammatory processinflammatory process
over obturator muscleover obturator muscle
– Ruptured appendixRuptured appendix
– Pelvic abscessPelvic abscess
Assessment of the AbdomenAssessment of the Abdomen
BallottmentBallottment
• Used to displaceUsed to displace
excess fluid in theexcess fluid in the
abdominal cavity byabdominal cavity by
using stiffened fingersusing stiffened fingers
in a jabbing motionin a jabbing motion
• Determines a freeDetermines a free
floating massfloating mass
• If pain is associatedIf pain is associated
with an inflammatorywith an inflammatory
processprocess
Assessment of the AbdomenAssessment of the Abdomen
Bladder PalpationBladder Palpation
• Using deep palpationUsing deep palpation
start at the symphisisstart at the symphisis
pubis and palpate uppubis and palpate up
• Nodular bladder --Nodular bladder --
malignancymalignancy
• Asymmetrical bladderAsymmetrical bladder
– tumor in the bladdertumor in the bladder
– abdominal tumorabdominal tumor
causing compressioncausing compression
Assessment of the AbdomenAssessment of the Abdomen
Percussion -- GeneralPercussion -- General
• Visualize all organs as youVisualize all organs as you
percuss all quadrantspercuss all quadrants
• Tympanitic-Tympanitic- heard mostlyheard mostly
-- hollow organs.-- hollow organs.
• DullDull sound- over solidsound- over solid
organsorgans
• Dull sound in area whereDull sound in area where
it should be tympanitic --it should be tympanitic --
mass or tumor, pregnancy,mass or tumor, pregnancy,
ascitesascites
• Liver dullness- obliteratedLiver dullness- obliterated
in perforation.in perforation.
Assessment of the AbdomenAssessment of the Abdomen
Percussion -- Liver SpanPercussion -- Liver Span
• Percuss 7cm upward -- midclavicular line above thePercuss 7cm upward -- midclavicular line above the
umbilicus -- typany to dull -- markumbilicus -- typany to dull -- mark
• Percuss downward -- midclavicular line -- tympany toPercuss downward -- midclavicular line -- tympany to
dull - markdull - mark
• Normal - 6-12 cm in males & 10.5 cm in femalesNormal - 6-12 cm in males & 10.5 cm in females
Assessment of the AbdomenAssessment of the Abdomen
Percussion – Ascites (Shifting Dullness)Percussion – Ascites (Shifting Dullness)
• Percuss from above and below from dullness toPercuss from above and below from dullness to
tympany while patient on their backtympany while patient on their back
• Place patient on left side and percuss fromPlace patient on left side and percuss from
dullness to tympanydullness to tympany
Assessment of the AbdomenAssessment of the Abdomen
Percussion -- AscitesPercussion -- Ascites
• The umbilical area percusses dull because theThe umbilical area percusses dull because the
ascitic fluid pools in the dependent areaascitic fluid pools in the dependent area
• Ascites is present in cirrhosis and other liverAscites is present in cirrhosis and other liver
diseasesdiseases
Assessment of the AbdomenAssessment of the Abdomen
Fist Percussion -- KidneyFist Percussion -- Kidney
• Direct fist percussion -- strike the costovertebral angleDirect fist percussion -- strike the costovertebral angle
with one fistwith one fist
• Indirect fist percussion -- place palm overIndirect fist percussion -- place palm over
costovertebral angle and strike with other handcostovertebral angle and strike with other hand
• Pain indicative of infllammatory conditionPain indicative of infllammatory condition
Assessment of the AbdomenAssessment of the Abdomen
Fist Percussion -- LiverFist Percussion -- Liver
• Palm down RUQ --Palm down RUQ --
strike with other handstrike with other hand
• Tenderness can occurTenderness can occur
with:with:
– PyelonephritisPyelonephritis
– CholecystitisCholecystitis
– HepatitisHepatitis
Assessment of the AbdomenAssessment of the Abdomen
Percussion -- BladderPercussion -- Bladder
• Percuss from symphisis pubis (urinePercuss from symphisis pubis (urine
filled gives dull sound) - if patientfilled gives dull sound) - if patient
unable to empty it is secondary to:unable to empty it is secondary to:
– Neurogenic dysfuntionNeurogenic dysfuntion
– Benign prostatic hypertrophyBenign prostatic hypertrophy
– Post-op casePost-op case
– Urethral strictureUrethral stricture
– Some medicationsSome medications
ASSESSMENT OF THE ABDOMENASSESSMENT OF THE ABDOMEN
AUSCULTATIONAUSCULTATION
• Use diaphragm of stethoscopeUse diaphragm of stethoscope
lightly placedlightly placed
• RLQ->RUQ->LUQ->LLQRLQ->RUQ->LUQ->LLQ
• Bowel sounds 5-30/minBowel sounds 5-30/min
• High-pitched always heardHigh-pitched always heard
RLQ-ileo-ceacal areaRLQ-ileo-ceacal area
• Borborygmi -- “stomachBorborygmi -- “stomach
growling” -- normal,growling” -- normal,
hyperactive, gurgling soundhyperactive, gurgling sound
BOWEL SOUNDSBOWEL SOUNDS
• Bowel Sounds are high pitched sounds & are classifiedBowel Sounds are high pitched sounds & are classified
into 4 categoriesinto 4 categories
• 1.Normal1.Normal
• 2.Hypoactive2.Hypoactive
• 3.Absent3.Absent
• 4.Hyperactive4.Hyperactive
Normal Bowel SoundsNormal Bowel Sounds
• Normal Bowel Sounds areNormal Bowel Sounds are
high pitched with Clicks andhigh pitched with Clicks and
Gargles , which occur every 5Gargles , which occur every 5
to 15 seconds intervalsto 15 seconds intervals
HYPOACTIVE BOWEL SOUNDSHYPOACTIVE BOWEL SOUNDS
ABSENT BOWEL SOUNDSABSENT BOWEL SOUNDS
HYPERACTIVE BOWEL SOUNDSHYPERACTIVE BOWEL SOUNDS
ASSESSMENT OF THE ABDOMENASSESSMENT OF THE ABDOMEN
AUSCULTATION -- BOWEL SOUNDSAUSCULTATION -- BOWEL SOUNDS
• Absent bowel sounds -- no sound for 4-5minutes--Absent bowel sounds -- no sound for 4-5minutes--
Late intestinal obstructionLate intestinal obstruction
• Mechanical obstruction -- adhesions, hernias,Mechanical obstruction -- adhesions, hernias,
massesmasses
• Non-mechanical obstruction -- no intestinalNon-mechanical obstruction -- no intestinal
contraction (paralytic ileus) -- physiological,contraction (paralytic ileus) -- physiological,
neurogenic, and chemical imbalancesneurogenic, and chemical imbalances
Assessment of the AbdomenAssessment of the Abdomen
Auscultation -- Bowel SoundsAuscultation -- Bowel Sounds
• Hypoactive bowel sounds -- indicatesHypoactive bowel sounds -- indicates
decreased motilitydecreased motility
– PeritonitisPeritonitis
– Non-mechanical obstructionNon-mechanical obstruction
– InflammationInflammation
– GangreneGangrene
– Electrolyte imbalancesElectrolyte imbalances
– Intraoperative manipulation of theIntraoperative manipulation of the
bowelbowel
Assessment of the AbdomenAssessment of the Abdomen
Auscultation -- Bowel SoundsAuscultation -- Bowel Sounds
• Hyperactive bowel sounds -- increasedHyperactive bowel sounds -- increased
motility of the bowelmotility of the bowel
– GastroenteritisGastroenteritis
– DiarrheaDiarrhea
– Laxative useLaxative use
– Subsiding ileusSubsiding ileus
Assessment of the AbdomenAssessment of the Abdomen
Auscultation -- Bowel SoundsAuscultation -- Bowel Sounds
• High pitched tinkling hyperactive bowelHigh pitched tinkling hyperactive bowel
soundssounds
– Caused by powerful peristaltic actionCaused by powerful peristaltic action
indicative of partial obstructionindicative of partial obstruction
– Abdominal crampingAbdominal cramping
Assessment of the AbdomenAssessment of the Abdomen
Auscultation -- Venous humAuscultation -- Venous hum
• A continuous pulsing or fibrillary soundA continuous pulsing or fibrillary sound
• If present in the periumbilical area isIf present in the periumbilical area is
secondary to portal vein obstructionsecondary to portal vein obstruction
• Portal hypertension caused by cirrhosisPortal hypertension caused by cirrhosis
Assessment of the AbdomenAssessment of the Abdomen
Auscultation -- Friction RubsAuscultation -- Friction Rubs
• Using the diaphragm of the stethoscopeUsing the diaphragm of the stethoscope
a sound similar to rubbing sandpapera sound similar to rubbing sandpaper
togethertogether
• Sound increases with inspirationSound increases with inspiration
– TumorsTumors
– InflammationInflammation
– InfarctionInfarction
Assessment of the AbdomenAssessment of the Abdomen
Other Examination:Other Examination:
• Hernia ExaminationHernia Examination
• Pelvic ExaminationPelvic Examination
• Rectal examinationRectal examination
• Laboratory ExaminationLaboratory Examination
• Diagnostic ImagingDiagnostic Imaging
Acute AbdomenAcute Abdomen
Associated symptoms.Associated symptoms.
ECG: inferior wall MI may present withECG: inferior wall MI may present with
epigastric or upper abdominal painepigastric or upper abdominal pain
with no clear disorder identifiedwith no clear disorder identified
Identify conditions for urgent surgery:Identify conditions for urgent surgery:
hypotension without GI bleeding,hypotension without GI bleeding,
aneurysm, rigid abdomen in a pt withaneurysm, rigid abdomen in a pt with
abdominal pain,.abdominal pain,.
Acute abdomenAcute abdomen
Treat shock.Treat shock.
Determine shock severity:Determine shock severity:
1) compensated shock1) compensated shock
2) decompensated shock2) decompensated shock
Acute abdomenAcute abdomen
• -First Understand the-First Understand the
• -Anatomic correlation of abdominal pain-Anatomic correlation of abdominal pain
- Types of abdominal pain:- Types of abdominal pain:
-Causes of abdominal pain by quadrants,-Causes of abdominal pain by quadrants,
-Parietal pain, visceral pain and referred pain.-Parietal pain, visceral pain and referred pain.
-Most common surgical Causes of-Most common surgical Causes of RUQ:RUQ:
abdominal painabdominal pain
• -Perforated duodenal ulcer-Perforated duodenal ulcer
• -Cholecystitis-Cholecystitis
The Acute AbdomenThe Acute Abdomen
Rapid Onset of PainRapid Onset of Pain
The Acute AbdomenThe Acute Abdomen
Slow Onset of PainSlow Onset of Pain
Acute AbdomenAcute Abdomen
• Hepatic abscessHepatic abscess
• Retrocecal appendicitisRetrocecal appendicitis
• Appendicitis in pregnant womanAppendicitis in pregnant woman
RLQ:RLQ:
• AppendicitisAppendicitis
• Cecal diverticulitisCecal diverticulitis
• Meckel’s diverticulitisMeckel’s diverticulitis
Acute abdomenAcute abdomen
LLQ:LLQ:
• Splenic ruptureSplenic rupture
• Splenic abscessSplenic abscess
LUQ:LUQ:
• Splenic ruptureSplenic rupture
• Splenic abscessSplenic abscess
Diffuse pain:Diffuse pain:
• Bowel obstructionBowel obstruction
• Leaking aneurismLeaking aneurism
• MesentericMesenteric
• IsquemiaIsquemia
Periumbilical:Periumbilical:
• Early appendicitisEarly appendicitis
• Referred pain fromReferred pain from
small bowelsmall bowel
Acute abdomenAcute abdomen
• NonSurgical causes of abdominal pain:NonSurgical causes of abdominal pain:
• RUQ:RUQ:
• (RLL) pneumonia(RLL) pneumonia
• Biliary colicBiliary colic
• CholangitisCholangitis
• HepatitisHepatitis
• Fitz-Hugh-Curtis SyndromeFitz-Hugh-Curtis Syndrome
(perihepatitis associated(perihepatitis associated
c chlamydial infection of cervix)c chlamydial infection of cervix)
Midepigastric:Midepigastric:
• PUD non perforatedPUD non perforated
• MIMI
• EsophagitisEsophagitis
• PEPE
• PancreatitisPancreatitis
• Herpes ZosterHerpes Zoster
• Rectus sheat hematomaRectus sheat hematoma
Acute abdomenAcute abdomen
• RLQ or LLQ:RLQ or LLQ:
• Ureteral calculiUreteral calculi
• Regional enteritisRegional enteritis
(Crohn’s Ds)(Crohn’s Ds)
• InflammatoryInflammatory
bowel Diseasebowel Disease
(Regional enteritis,(Regional enteritis,
UC)UC)
• PIDPID
• EndometriosisEndometriosis
• ProstatitisProstatitis
• MittelschmerzMittelschmerz
• UTIUTI
• Ruptured ovarianRuptured ovarian
cystcyst
• LUQ:LUQ:
• LLL pneumoniaLLL pneumonia
• Gastritis,Gastritis,
• SplenomegalySplenomegaly
Acute AbdomenAcute Abdomen
Diffuse:Diffuse:
• Abdominal wallAbdominal wall
hematomahematoma
• Spider biteSpider bite
• Lead poisoningLead poisoning
• Addisonian crisisAddisonian crisis
• Sickle cell crisisSickle cell crisis
• Diabetic ketoacidosisDiabetic ketoacidosis
(DKA)(DKA)
• DiabeticDiabetic
gastropathygastropathy
• Opiate withdrawalOpiate withdrawal
GI emergenciesGI emergencies• AppendicitisAppendicitis
• AscitesAscites
• Biliary ColicBiliary Colic
• Boerhaave SyndromeBoerhaave Syndrome
• CholangitisCholangitis
• Cholecystitis, AcuteCholecystitis, Acute
• CirrhosisCirrhosis
• Colitis, IschemicColitis, Ischemic
• Colitis, UlcerativeColitis, Ulcerative
• Crohn’s DiseaseCrohn’s Disease
• DiverticulitisDiverticulitis
• DiverticulosisDiverticulosis
• GastritisGastritis
• GastroenteritisGastroenteritis
• Gastroesophageal RefluxGastroesophageal Reflux
DiseaseDisease
• HemorrhoidsHemorrhoids
• Hepatic AbscessHepatic Abscess
• Hepatic EncephalopathyHepatic Encephalopathy
• Hepatitis, AlcoholicHepatitis, Alcoholic
• Hepatitis, ViralHepatitis, Viral
• Incarcerated HerniaIncarcerated Hernia
• Intestinal ObstructionIntestinal Obstruction
• Irritable Bowel SyndromeIrritable Bowel Syndrome
• Mallory-Weiss SyndromeMallory-Weiss Syndrome
• Meckel DiverticulumMeckel Diverticulum
• Pancreatitis, AcutePancreatitis, Acute
• Peptic Ulcer DiseasePeptic Ulcer Disease
• Perforated peptic ulcerPerforated peptic ulcer
• Variceal BleedingVariceal Bleeding
Acute AbdomenAcute Abdomen
Suprapubic:Suprapubic:
• Ectopic pregnancyEctopic pregnancy
• Ovarian torsionOvarian torsion
• Tubo-ovarian abscessTubo-ovarian abscess
• Incarcerated groin herniaIncarcerated groin hernia
Which of the following is the MOSTWhich of the following is the MOST
common cause of painful rectal bleeding?common cause of painful rectal bleeding?
a. internal hemorrhoida. internal hemorrhoid
b. external hemorrhoidb. external hemorrhoid
c. diverticulitisc. diverticulitis
d. anal fissured. anal fissure
e. rectal foreign bodye. rectal foreign body
QUESTIONSQUESTIONS
ANDAND
ANSWERS SESSIONANSWERS SESSION
Plain xray of abdomenPlain xray of abdomen
ERCP Film showing a hypoecheoicERCP Film showing a hypoecheoic
shadowshadow
Plain xray chest showsPlain xray chest shows
Plain xray abdomen in errect posturePlain xray abdomen in errect posture
Plain xray abdomen in errect posturePlain xray abdomen in errect posture
AnswerAnswer
Intestinal ObstructionIntestinal Obstruction
due to an Adhesivedue to an Adhesive
BandBand
AnswerAnswer
Intestinal ObstructionIntestinal Obstruction
ANSWERANSWER
Intestinal ObstructionIntestinal Obstruction
Gallstone IleusGallstone Ileus
AnswerAnswer
» Intestinal Obstruction-Intestinal Obstruction-
following obstructedfollowing obstructed
Umbilical HerniaUmbilical Hernia
AnswerAnswer
• Intestinal ObstructionIntestinal Obstruction
Incarcerated &Incarcerated &
Strangulating &Strangulating &
gangrenous umbilicalgangrenous umbilical
Hernia at laparotomyHernia at laparotomy
AnswerAnswer
• Intestinal ObstructionIntestinal Obstruction
Following an incisional HerniaFollowing an incisional Hernia
with Incarceration andwith Incarceration and
StrangulationStrangulation
AnswerAnswer
• Intestinal obstructionIntestinal obstruction
alongwith gangrene ofalongwith gangrene of
the small bowelthe small bowel
following an incarceratedfollowing an incarcerated
ventral herniaventral hernia
A. Anal fissures result from a linear tear of theA. Anal fissures result from a linear tear of the
anal canal beginning at or just below theanal canal beginning at or just below the
dentate line and extending distally along thedentate line and extending distally along the
anal canal.anal canal.
B. Pt’s complain of sharp, cutting pain, mostB. Pt’s complain of sharp, cutting pain, most
severe during and immediately after a bowelsevere during and immediately after a bowel
movement.movement.
C. Bleeding is scant and bright red.C. Bleeding is scant and bright red.
D. Anal fissures are especially painful b/c of theD. Anal fissures are especially painful b/c of the
rich supply of somatic sensory nerve fibersrich supply of somatic sensory nerve fibers
located in the anodermlocated in the anoderm
All of the following are true regarding acalculousAll of the following are true regarding acalculous
cholecystitis EXCEPTcholecystitis EXCEPT
A. It occurs in 5-10% of pt’s with acuteA. It occurs in 5-10% of pt’s with acute
cholecystitischolecystitis
B. Pt’s are frequently elderly and have aB. Pt’s are frequently elderly and have a
history of DMhistory of DM
C. It often occurs as a complication of anotherC. It often occurs as a complication of another
processprocess
D. Diagnosis is difficult due to the subtleD. Diagnosis is difficult due to the subtle
clinical presentationclinical presentation
E. Gallstones are absentE. Gallstones are absent
Answer DAnswer D
Answer DAnswer D
• A calculus cholecystitis occurs in 5-10% of pt’sA calculus cholecystitis occurs in 5-10% of pt’s
c cholecystitis.c cholecystitis.
• Pts frequently are elderly and have h/o DMPts frequently are elderly and have h/o DM
• There are 2 distinguishing features ofThere are 2 distinguishing features of
acalculous cholecysititisacalculous cholecysititis
– 1. it frequently occurs as a complication of1. it frequently occurs as a complication of
another processanother process
– 2. pts frequently are gravely ill on initial2. pts frequently are gravely ill on initial
presentationpresentation
Which of the following drugs is NOTWhich of the following drugs is NOT
associated with acute pancreatitis?associated with acute pancreatitis?
a. Heparina. Heparin
b. Furosemideb. Furosemide
c. Rifampinc. Rifampin
d. Salicylatesd. Salicylates
e. Warfarine. Warfarin
Answer AAnswer A
• Drugs and toxins are major causes ofDrugs and toxins are major causes of
acute pancreatitis.acute pancreatitis.
• Some of the meds assoc c the occurrenceSome of the meds assoc c the occurrence
are OCP’s, glucocorticoids, rifampin,are OCP’s, glucocorticoids, rifampin,
tetracycline, isoniazid, thiazide diuretics,tetracycline, isoniazid, thiazide diuretics,
furosemide, salicylates, indomethacin,furosemide, salicylates, indomethacin,
calcium, warfarin, and acetominophen.calcium, warfarin, and acetominophen.
• Other etiologic factors contributingOther etiologic factors contributing
include infection, collagen vascular ds’s,include infection, collagen vascular ds’s,
metabolic disturbances, and traumametabolic disturbances, and trauma
In working up a patient with acute abdominal pain,In working up a patient with acute abdominal pain,
which of the following etiologies is LEAST likelywhich of the following etiologies is LEAST likely
to represent an immediate life threat?to represent an immediate life threat?
a. Myocardial infarctiona. Myocardial infarction
b. Splenic ruptureb. Splenic rupture
c. Abdominal Aortic Aneurysmc. Abdominal Aortic Aneurysm
d. Perforated duodenal ulcerd. Perforated duodenal ulcer
e. Ruptured ectopic pregnancye. Ruptured ectopic pregnancy
Answer DAnswer D
• When approaching a pt c acute abd pain, the clinicianWhen approaching a pt c acute abd pain, the clinician
must consider conditions that can be an immediatemust consider conditions that can be an immediate
threat to the patient’s life.threat to the patient’s life.
• Splenic rupture, ruptured ectopic pregn, and AAA canSplenic rupture, ruptured ectopic pregn, and AAA can
all be associated with massive bleeding and rapidall be associated with massive bleeding and rapid
decline.decline.
• Extra abdominal conditions that present with abd painExtra abdominal conditions that present with abd pain
such as MI can also be life threatening.such as MI can also be life threatening.
• Perforated duodenal ulcer are serious but almost neverPerforated duodenal ulcer are serious but almost never
result in significant hemorrhage, and thus are notresult in significant hemorrhage, and thus are not
usually an immediate threat to life.usually an immediate threat to life.
Which of the following statements is TRUEWhich of the following statements is TRUE
regarding acute abdominal pain?regarding acute abdominal pain?
A. Peritonitis causes visceral type of pain and isA. Peritonitis causes visceral type of pain and is
secondary to peritoneal inflammation from an irritantsecondary to peritoneal inflammation from an irritant
B. Obstruction of a hollow viscus produces colicky,B. Obstruction of a hollow viscus produces colicky,
diffuse visceral pain assoc with N/Vdiffuse visceral pain assoc with N/V
C. Intra abdominal causes of pain include bacterialC. Intra abdominal causes of pain include bacterial
peritonitis, bowel ischemia, and tub ovarian abscessperitonitis, bowel ischemia, and tub ovarian abscess
D. referred pain from the abdomen may radiate to theD. referred pain from the abdomen may radiate to the
back or groin, but not into the thoraxback or groin, but not into the thorax
E. Metabolic disorders are rarely a significant sourceE. Metabolic disorders are rarely a significant source
of acute abdominal painof acute abdominal pain
Answer BAnswer B
Three types of pain responses are possible with acute abd. painThree types of pain responses are possible with acute abd. pain
 Peritonitis is a somatic pain and is usually sharper, morePeritonitis is a somatic pain and is usually sharper, more
 constant, and more localized than visceral pain.constant, and more localized than visceral pain.
 Obstruction of a hollow viscus is a common cause of visceralObstruction of a hollow viscus is a common cause of visceral
 pain and is colicky, intermittent, and usually mid-line.pain and is colicky, intermittent, and usually mid-line.
 Referred pain is often felt in the back, groin, or thighs. Pt’sReferred pain is often felt in the back, groin, or thighs. Pt’s
 may also c/o pain in the supraclavicular region especially ifmay also c/o pain in the supraclavicular region especially if
 the diaphragm is irritated by collections of blood or pus.the diaphragm is irritated by collections of blood or pus.
 Abd pain can arise from intraabd, extraabd, metabolic, orAbd pain can arise from intraabd, extraabd, metabolic, or
neurogenic origins.neurogenic origins.
Intrabdominal origins of pain are divided intoIntrabdominal origins of pain are divided into
3 categories:3 categories:
– Peritoneal inflame, obstruction of a hollowPeritoneal inflame, obstruction of a hollow
viscus, and vascular etiologies.viscus, and vascular etiologies.
– Extraabd sources can arise from the abd wall,Extraabd sources can arise from the abd wall,
thorax, or pelvis (as in the case of tubo-ovarianthorax, or pelvis (as in the case of tubo-ovarian
abscess)abscess)
– Metabolic disorders such as DKA and Sickle cellMetabolic disorders such as DKA and Sickle cell
crisis often present with diffuse abd paincrisis often present with diffuse abd pain
All of the following are TRUE regarding theAll of the following are TRUE regarding the
evaluation of a patient with acute abdominalevaluation of a patient with acute abdominal
pain EXCEPTpain EXCEPT
AA. The onset, location, and severity of pain are. The onset, location, and severity of pain are
useful differentiating factorsuseful differentiating factors
B. The most important physical examinationB. The most important physical examination
modality is palpationmodality is palpation
C. The WBC may be normal even in inflammatoryC. The WBC may be normal even in inflammatory
conditions such as appendicitisconditions such as appendicitis
D. Ultra-sonography is a valuable imaging toolD. Ultra-sonography is a valuable imaging tool
increasingly availableincreasingly available
E. Analgesic medications should be withheld until aE. Analgesic medications should be withheld until a
surgeon evaluates the patient because they may obscuresurgeon evaluates the patient because they may obscure
the diagnosis.the diagnosis.
Answer EAnswer E
• The evaluation of abd pain should begin with a detailed hx.The evaluation of abd pain should begin with a detailed hx.
• The onset, severity, location, and character of pain and theThe onset, severity, location, and character of pain and the
presence of associated symptoms guide work-up and tx.presence of associated symptoms guide work-up and tx.
• Although a complete PE is necessary, palpation of the abd isAlthough a complete PE is necessary, palpation of the abd is
the most important modality for dx.the most important modality for dx.
• Lab tests are useful adjuncts, but the limitations of a CBCLab tests are useful adjuncts, but the limitations of a CBC
must be recognized.must be recognized.
• Helpful imaging modalities include standard XR, U/S,Helpful imaging modalities include standard XR, U/S,
barium contrast studies, and CTbarium contrast studies, and CT
• IV opiate analgesia is humane and may actually assist in dxIV opiate analgesia is humane and may actually assist in dx
by facilitating PE in a pt who could otherwise not tolerate it.by facilitating PE in a pt who could otherwise not tolerate it.
Which of the following is the most commonWhich of the following is the most common
cause of upper GI bleeding?cause of upper GI bleeding?
A. Esophageal varicesA. Esophageal varices
B. Mallory-Weiss tearB. Mallory-Weiss tear
C. PUDC. PUD
D. Erosive gastritisD. Erosive gastritis
E. Arteriovenous malformationsE. Arteriovenous malformations
Answer CAnswer C
• Upper GI bleeding is defined as bleeding that originates proximal to theUpper GI bleeding is defined as bleeding that originates proximal to the
ligament of Treitz.ligament of Treitz.
• PUD, including gastric, duodenal, and stomachal ulcers, is the MC cause ofPUD, including gastric, duodenal, and stomachal ulcers, is the MC cause of
upper GI bleeding (60%)upper GI bleeding (60%)
• The next MC are erosive gastritis, esophagitis, and duodenitis (15%)The next MC are erosive gastritis, esophagitis, and duodenitis (15%)
• Gastric irritants such as ETOH, salicylates, and NSAIDS, predispose pts toGastric irritants such as ETOH, salicylates, and NSAIDS, predispose pts to
upper GI bleeding.upper GI bleeding.
• Varices (only 6%) from portal HTN in Ethers carry a high mortality rate.Varices (only 6%) from portal HTN in Ethers carry a high mortality rate.
• Mallory-Weiss syndrome is due to a mucosal tear in the esophagus and isMallory-Weiss syndrome is due to a mucosal tear in the esophagus and is
classically assoc c repeated bouts of retching.classically assoc c repeated bouts of retching.
• AV malformations are an uncommon cause of upper GI bleedAV malformations are an uncommon cause of upper GI bleed
A Pt presents with what appears to be a massiveA Pt presents with what appears to be a massive
lower GI hemorrhage. Which one of the followinglower GI hemorrhage. Which one of the following
is the LEAST likely etiology?is the LEAST likely etiology?
A. DiverticulosisA. Diverticulosis
B. AngiodysplasiaB. Angiodysplasia
C. Gastric varicesC. Gastric varices
D. Duodenal ulcerD. Duodenal ulcer
E. HaemorrhoidsE. Haemorrhoids
Answer EAnswer E
• The MC cause of what initially appears to be lower GI bleeding isThe MC cause of what initially appears to be lower GI bleeding is
actually bleeding from an upper GI source.actually bleeding from an upper GI source.
• Brisk bleeding from either varices or PUD can be the cause ofBrisk bleeding from either varices or PUD can be the cause of
apparent lower GI hemorrhage.apparent lower GI hemorrhage.
• Diverticulosis and angiodysplasia are the MC causes of confirmedDiverticulosis and angiodysplasia are the MC causes of confirmed
lower GI bleed.lower GI bleed.
Both occur more commonly in elderly, are painless, and may beBoth occur more commonly in elderly, are painless, and may be
massive.massive.
• Although hemorrhoids are a common etiology of minor lower GIAlthough hemorrhoids are a common etiology of minor lower GI
bleed, usually not significant hemorrhagebleed, usually not significant hemorrhage
• Other less frequent sources of lower GI bleed include malignancies,Other less frequent sources of lower GI bleed include malignancies,
IBD, polyps, infectious gastroenteritis, and Meckel’s diverticulum.IBD, polyps, infectious gastroenteritis, and Meckel’s diverticulum.
Which of the following scenarios may representWhich of the following scenarios may represent
acute appendicitis?acute appendicitis?
A. a 4 y/o male with vomiting and lethargyA. a 4 y/o male with vomiting and lethargy
B. a 75 y/o female with fever and abdominal painB. a 75 y/o female with fever and abdominal pain
C. a 26 y/o female who is 32 weeks pregnant withC. a 26 y/o female who is 32 weeks pregnant with
right upper quadrant painright upper quadrant pain
D. a 45 y/o male with AIDS and who hasD. a 45 y/o male with AIDS and who has
vomiting and diarrheavomiting and diarrhea
E. All of the aboveE. All of the above
Answer EAnswer E
• Certain groups of pts have atypical presentations and are at risk forCertain groups of pts have atypical presentations and are at risk for
delayed dx of acute appendicitis.delayed dx of acute appendicitis.
• Children <6 y/o = 57% misdiagnosed; 90% perforation ratesChildren <6 y/o = 57% misdiagnosed; 90% perforation rates
• Elderly pts, subtle sx, & high perforationsElderly pts, subtle sx, & high perforations
• Pregnant pts pose difficulty b/c the gravid uterus changes thePregnant pts pose difficulty b/c the gravid uterus changes the
position of the appendixposition of the appendix
• An U/S can aid in distinguishing pelvic vs abd pathologyAn U/S can aid in distinguishing pelvic vs abd pathology
• Immunocompromised are susceptible to delayed dx b/c of theirImmunocompromised are susceptible to delayed dx b/c of their
frequent unrelated GI sx.frequent unrelated GI sx.
• CT is helpful in differentiating surgical from nonsurgical conditionsCT is helpful in differentiating surgical from nonsurgical conditions
What is the most common cause ofWhat is the most common cause of
large bowel obstruction?large bowel obstruction?
A. AdhesionsA. Adhesions
B. Incarcerated herniaB. Incarcerated hernia
C. DiverticulitisC. Diverticulitis
D. NeoplasmD. Neoplasm
E. Sigmoid volvulusE. Sigmoid volvulus
Answer DAnswer D
• It is important to distinguish between largeIt is important to distinguish between large
and small bowel obstruction b/c tx differsand small bowel obstruction b/c tx differs
• The MC cause of colonic obstruction is neoplasmThe MC cause of colonic obstruction is neoplasm
• 22ndnd
MC = diverticulitis, followed by sigmoidMC = diverticulitis, followed by sigmoid
volvulusvolvulus
• MC SBO = surgical adhesionsMC SBO = surgical adhesions
• 22ndnd
MC SBO = hernias and primary smallMC SBO = hernias and primary small
bowel lesionsbowel lesions
A 40 y/o female with known gallstones presents with colickyA 40 y/o female with known gallstones presents with colicky
RUQ pain and vomiting. She has a h/o similar episodes thatRUQ pain and vomiting. She has a h/o similar episodes that
usually resolve after 3-4 hrs. VS: BP 110/60, P 78, R 16, Tusually resolve after 3-4 hrs. VS: BP 110/60, P 78, R 16, T
98.4*F. PE: mildly tender RUQ without signs of peritonitis.98.4*F. PE: mildly tender RUQ without signs of peritonitis.
Which of the following would be LEAST appropriate in her EDWhich of the following would be LEAST appropriate in her ED
management?management?
a. IV fluidsa. IV fluids
b. Pain control c opiate analgesicsb. Pain control c opiate analgesics
c. pain control c ketorolacc. pain control c ketorolac
d. antiemetic administrationd. antiemetic administration
e. immediate surgical consultatione. immediate surgical consultation
Answer EAnswer E
• Pts with uncomplicated symptomatic cholelithiasis do notPts with uncomplicated symptomatic cholelithiasis do not
require immediate surgical intervention.require immediate surgical intervention.
• ED intervention is geared toward pain relief and correction ofED intervention is geared toward pain relief and correction of
volume deficitsvolume deficits
• Pain control can be achieved with administration of opiates orPain control can be achieved with administration of opiates or
ketorolacketorolac
• Antiemetics and gastric decompression with an NGT may beAntiemetics and gastric decompression with an NGT may be
necessary for tx of protracted vomiting.necessary for tx of protracted vomiting.
• If the pt’s sx resolve within 4-6 hrs and she tolerates oralIf the pt’s sx resolve within 4-6 hrs and she tolerates oral
fluids, D/C home with output f/u is appropriatefluids, D/C home with output f/u is appropriate
Which of the following is the MOST commonWhich of the following is the MOST common
presentation of gallstones?presentation of gallstones?
a. acute pancreatitisa. acute pancreatitis
b. acute cholecysitisb. acute cholecysitis
c. biliary colicc. biliary colic
d. ascending cholangitisd. ascending cholangitis
e. gallbladder empyemae. gallbladder empyema
Answer CAnswer C• Pt’s with gallstones present in a variety of ways, and biliaryPt’s with gallstones present in a variety of ways, and biliary
colic (or symptomatic cholecystitis) is the MC.colic (or symptomatic cholecystitis) is the MC.
• The pain is colicky in nature, occurs after meals, and typicallyThe pain is colicky in nature, occurs after meals, and typically
lasts from 1-6 hrs.lasts from 1-6 hrs.
• Pain lasting longer than 6 hrs that is accompanied by fever orPain lasting longer than 6 hrs that is accompanied by fever or
leukocytosis suggests cholecystitis.leukocytosis suggests cholecystitis.
• Biliary colic and acute cholecystitis are by far the MCBiliary colic and acute cholecystitis are by far the MC
manifestations of gallstone dsmanifestations of gallstone ds
• Complications of gallstones may be life threatening.Complications of gallstones may be life threatening.
• Acute pancreatitis, ascending cholangitis, gallbladderAcute pancreatitis, ascending cholangitis, gallbladder
empyema, and emphysematous cholecystitis all requireempyema, and emphysematous cholecystitis all require
aggressive pt resuscitation and prompt surgical consultaggressive pt resuscitation and prompt surgical consult
What is the MOST common cause of pancreatitisWhat is the MOST common cause of pancreatitis
in an urban hospital setting?in an urban hospital setting?
a. cholelithiasisa. cholelithiasis
b. alcoholismb. alcoholism
c. abdominal traumac. abdominal trauma
d. penetrating peptic ulcerd. penetrating peptic ulcer
e. salicylate poisoninge. salicylate poisoning
Answer BAnswer B
• Acute pancreatitis is a common cause of abdominalAcute pancreatitis is a common cause of abdominal
pain.pain.
• In the US cholelithiasis and alcoholism account forIn the US cholelithiasis and alcoholism account for
90%.90%.
• ETOH related ds is more common in the urbanETOH related ds is more common in the urban
setting, and typically affects males 35-45.setting, and typically affects males 35-45.
• Biliary ds is more frequent in the community hospitalBiliary ds is more frequent in the community hospital
setting and typically affects females >50.setting and typically affects females >50.
• After biliary and ETOH = drugs (1/2 of remaining cases)After biliary and ETOH = drugs (1/2 of remaining cases)
A 55 y/o female presents to the ED with a feverA 55 y/o female presents to the ED with a fever
4 days after undergoing a laparscopic4 days after undergoing a laparscopic
cholecystectomy. What is the MOST likelycholecystectomy. What is the MOST likely
cause of the fever?cause of the fever?
a. Pneumoniaa. Pneumonia
b. Thrombophlebitisb. Thrombophlebitis
c. Urinary Tract infectionc. Urinary Tract infection
d. Wound infectiond. Wound infection
e. Deep venous thrombosise. Deep venous thrombosis
Answer CAnswer C• Laparoscopic procedures and early postsurgicalLaparoscopic procedures and early postsurgical
discharge are becoming increasingly commondischarge are becoming increasingly common
cost-effective alternatives to laparotomy.cost-effective alternatives to laparotomy.
• As a result, more pts are presenting to the EDAs a result, more pts are presenting to the ED
with postoperative fever.with postoperative fever.
• Fever postopFever postop
– <24 h = atelectasis or necrotizing strept<24 h = atelectasis or necrotizing strept
infectionsinfections
– 24-72 h = respiratory (pneumonia), IV catheter24-72 h = respiratory (pneumonia), IV catheter
complications (thrombophlebitis)complications (thrombophlebitis)
– 3-5 d = UTI’s (MC in female, and pts c cath’s)3-5 d = UTI’s (MC in female, and pts c cath’s)
– 7-10 d = wound infections7-10 d = wound infections
– DVT’s can result in fever @ any time, pero usually >5dDVT’s can result in fever @ any time, pero usually >5d
A pt with suspected cholelithiasis presents to the ED.A pt with suspected cholelithiasis presents to the ED.
What is the initial imaging study of choice?What is the initial imaging study of choice?
a. abdominal plain filma. abdominal plain film
b. abdominal ultrasoundb. abdominal ultrasound
c. abdominal CTc. abdominal CT
d. Radionuclide scan (HIDA)d. Radionuclide scan (HIDA)
e. Barium contrast radiographye. Barium contrast radiography
Answer BAnswer B
• U/S has emerged as a valuable tool for certain conditions in the ED.U/S has emerged as a valuable tool for certain conditions in the ED.
• It is the initial study of choice for eval of pts with RUQ pain,It is the initial study of choice for eval of pts with RUQ pain,
and can accurately detect cholelithiasis.and can accurately detect cholelithiasis.
• Plain film is a poor imaging choice to detect gallstones (onlyPlain film is a poor imaging choice to detect gallstones (only
15%), but is useful in evaluating obstruction or suspected perforation.15%), but is useful in evaluating obstruction or suspected perforation.
• CT is the diagnostic tool of choice for many abdominal conditionsCT is the diagnostic tool of choice for many abdominal conditions
including pancreatitis, some trauma, and selected AAA, but is moreincluding pancreatitis, some trauma, and selected AAA, but is more
costly and invasive than U/S for evaluating gallstones.costly and invasive than U/S for evaluating gallstones.
• HIDA scan is a useful adjunct if U/S results are inconclusive orHIDA scan is a useful adjunct if U/S results are inconclusive or
acalculous cholecysititis is suspected.acalculous cholecysititis is suspected.
• Barium studies are useful for imaging in some GI conditions, especiallyBarium studies are useful for imaging in some GI conditions, especially
suspected intussusception, but not for eval of GBsuspected intussusception, but not for eval of GB
Which of the following diagnostic study is mostWhich of the following diagnostic study is most
useful in the evaluation of a patient in the acuteuseful in the evaluation of a patient in the acute
stages of diverticulitis?stages of diverticulitis?
a. CT scana. CT scan
b. barium enemab. barium enema
c. ultrasound of the abdomenc. ultrasound of the abdomen
d. colonoscopyd. colonoscopy
e. sigmoidoscopye. sigmoidoscopy
Answer AAnswer A
• Both endoscopy and barium enema areBoth endoscopy and barium enema are
contraindicated in pts during acute stages ofcontraindicated in pts during acute stages of
diverticulitis b/c of risk of perforation.diverticulitis b/c of risk of perforation.
A 28 y/o man has complaints of intermittent, colicky,A 28 y/o man has complaints of intermittent, colicky,
periumbilical, and lower-quadrant pain for 24 hours. Theperiumbilical, and lower-quadrant pain for 24 hours. The
patient admits to nausea and decreased appetite. He is afebrile.patient admits to nausea and decreased appetite. He is afebrile.
Which of the following is the most likely diagnosis?Which of the following is the most likely diagnosis?
a. acute appendicitisa. acute appendicitis
b. acute pancreatitisb. acute pancreatitis
c. pyelonephritisc. pyelonephritis
d. gastroenteritisd. gastroenteritis
e. peptic ulcer diseasee. peptic ulcer disease
Answer DAnswer D
• The pain pattern is most consistent with a dx ofThe pain pattern is most consistent with a dx of
gastroenteritis.gastroenteritis.
• Acute appendicitis typically causes periumbilical pain thatAcute appendicitis typically causes periumbilical pain that
migrates to the RLQmigrates to the RLQ
• Acute pancreatitis radiates to the back or shoulderAcute pancreatitis radiates to the back or shoulder
• Pyelonephritis is “loin to groin”Pyelonephritis is “loin to groin”
• PUD is typically located in the epigastrumPUD is typically located in the epigastrum
A 7 y/o boy presents with c/o flank pain, fever,A 7 y/o boy presents with c/o flank pain, fever,
frequency, dysuria, and hematuria for 1 day. The UAfrequency, dysuria, and hematuria for 1 day. The UA
shows >10 WBC’s per high-powered field, RBC’s, andshows >10 WBC’s per high-powered field, RBC’s, and
WBC casts. Of the following, the most likelyWBC casts. Of the following, the most likely
diagnosis is:diagnosis is:
a. pyelonephritisa. pyelonephritis
b. acute cystitisb. acute cystitis
c. urethritisc. urethritis
d. renal calculid. renal calculi
e. urinary incontinencee. urinary incontinence
Answer AAnswer A
• Pyelonephritis is an infection of the renal parenchyma,Pyelonephritis is an infection of the renal parenchyma,
accompanied by systemic sx such as fever, N/V and assocaccompanied by systemic sx such as fever, N/V and assoc
with WBC casts in the urine.with WBC casts in the urine.
• In pediatric male pt, its occurrence would warrantIn pediatric male pt, its occurrence would warrant
additional eval to r/o anatomic abnormalities in theadditional eval to r/o anatomic abnormalities in the
urinary tracturinary tract
The most common cause of acute renal failure is :The most common cause of acute renal failure is :
a. prerenal azotemiaa. prerenal azotemia
b. acute parenchymal renal failureb. acute parenchymal renal failure
c. exogenous nephrotoxinsc. exogenous nephrotoxins
d. obstructive uropathyd. obstructive uropathy
e. rhabdomyolysise. rhabdomyolysis
Answer AAnswer A
• Prerenal azotemia results from renal hypoperfusionPrerenal azotemia results from renal hypoperfusion
and can be reversed upon restoration of blood flow.and can be reversed upon restoration of blood flow.
• It is not associated with structural damage to theIt is not associated with structural damage to the
kidney and is the most common cause of acute renalkidney and is the most common cause of acute renal
failure.failure.
y/o woman with amenorrhea is seen for vaginaly/o woman with amenorrhea is seen for vaginal
bleeding and abdominal pain. An ectopic pregnancy isbleeding and abdominal pain. An ectopic pregnancy is
suspected. Which of the following would support thesuspected. Which of the following would support the
suspicion?suspicion?
a. enlarged boggy uterusa. enlarged boggy uterus
b. ruptured fetal membranesb. ruptured fetal membranes
c. adnexal massc. adnexal mass
d. weak fetal heart beatd. weak fetal heart beat
e. painless profuse bleedinge. painless profuse bleeding
Answer CAnswer C
Classic features of an ectopic pregnancy are abdominal pain,Classic features of an ectopic pregnancy are abdominal pain,
bleeding, and adnexal mass in a pregnant woman.bleeding, and adnexal mass in a pregnant woman.

Mais conteúdo relacionado

Mais procurados

Approach to acute abdomen
Approach to acute abdomenApproach to acute abdomen
Approach to acute abdomenRunal Shah
 
Hemorrhoids:Its current management
Hemorrhoids:Its current managementHemorrhoids:Its current management
Hemorrhoids:Its current managementGeorge Mukoro
 
Acute cholecystitis case-based discussion
Acute cholecystitis case-based discussionAcute cholecystitis case-based discussion
Acute cholecystitis case-based discussionAbdullah Bin Eid
 
Acute appendicitis -Case Presentation
Acute appendicitis -Case PresentationAcute appendicitis -Case Presentation
Acute appendicitis -Case PresentationMohammed Aljaber
 
Case presentation: Chronic pancreatitis
Case presentation: Chronic pancreatitisCase presentation: Chronic pancreatitis
Case presentation: Chronic pancreatitisBSMMU
 
Case Write Up Surgical Gastric Carcinoma
Case Write Up Surgical Gastric CarcinomaCase Write Up Surgical Gastric Carcinoma
Case Write Up Surgical Gastric CarcinomaDr Slayer
 
Approach to a case of pain abdomen
Approach to a case of pain abdomenApproach to a case of pain abdomen
Approach to a case of pain abdomenRahul Arya
 
Intestinal obstruction
Intestinal obstructionIntestinal obstruction
Intestinal obstructionBISHAL SAPKOTA
 
Approach to Acute Abdominal Pain (in emergency ward)
Approach to Acute Abdominal Pain (in emergency ward)Approach to Acute Abdominal Pain (in emergency ward)
Approach to Acute Abdominal Pain (in emergency ward)niraj phoju
 
Appendicitis
AppendicitisAppendicitis
Appendicitiskr
 
Approach to abdominal pain
Approach to abdominal painApproach to abdominal pain
Approach to abdominal painZaheen Zehra
 
UG CASE PRESENTATION ON INGUINAL HERNIA
UG CASE PRESENTATION ON INGUINAL HERNIAUG CASE PRESENTATION ON INGUINAL HERNIA
UG CASE PRESENTATION ON INGUINAL HERNIAAyesha Huma
 
Benign Prostatic Hyperplasia (Surgical Case Presentation)
Benign Prostatic Hyperplasia (Surgical Case Presentation)Benign Prostatic Hyperplasia (Surgical Case Presentation)
Benign Prostatic Hyperplasia (Surgical Case Presentation)Dr. Aryan (Anish Dhakal)
 
Investigations in the case of abdominal pain
Investigations in the case of abdominal painInvestigations in the case of abdominal pain
Investigations in the case of abdominal painAbino David
 

Mais procurados (20)

Acute abdomen
Acute abdomenAcute abdomen
Acute abdomen
 
Approach to acute abdomen
Approach to acute abdomenApproach to acute abdomen
Approach to acute abdomen
 
Hemorrhoids:Its current management
Hemorrhoids:Its current managementHemorrhoids:Its current management
Hemorrhoids:Its current management
 
Acute cholecystitis case-based discussion
Acute cholecystitis case-based discussionAcute cholecystitis case-based discussion
Acute cholecystitis case-based discussion
 
Acute appendicitis -Case Presentation
Acute appendicitis -Case PresentationAcute appendicitis -Case Presentation
Acute appendicitis -Case Presentation
 
Case presentation: Chronic pancreatitis
Case presentation: Chronic pancreatitisCase presentation: Chronic pancreatitis
Case presentation: Chronic pancreatitis
 
Acute abdomen
Acute abdomenAcute abdomen
Acute abdomen
 
Case Write Up Surgical Gastric Carcinoma
Case Write Up Surgical Gastric CarcinomaCase Write Up Surgical Gastric Carcinoma
Case Write Up Surgical Gastric Carcinoma
 
Approach to a case of pain abdomen
Approach to a case of pain abdomenApproach to a case of pain abdomen
Approach to a case of pain abdomen
 
Intestinal obstruction
Intestinal obstructionIntestinal obstruction
Intestinal obstruction
 
Approach to Acute Abdominal Pain (in emergency ward)
Approach to Acute Abdominal Pain (in emergency ward)Approach to Acute Abdominal Pain (in emergency ward)
Approach to Acute Abdominal Pain (in emergency ward)
 
Hemorrhoids
HemorrhoidsHemorrhoids
Hemorrhoids
 
UTI Case Presentation
UTI Case PresentationUTI Case Presentation
UTI Case Presentation
 
Acute abdomen
Acute abdomenAcute abdomen
Acute abdomen
 
Appendicitis
AppendicitisAppendicitis
Appendicitis
 
Approach to abdominal pain
Approach to abdominal painApproach to abdominal pain
Approach to abdominal pain
 
UG CASE PRESENTATION ON INGUINAL HERNIA
UG CASE PRESENTATION ON INGUINAL HERNIAUG CASE PRESENTATION ON INGUINAL HERNIA
UG CASE PRESENTATION ON INGUINAL HERNIA
 
Acute abdomen
Acute abdomenAcute abdomen
Acute abdomen
 
Benign Prostatic Hyperplasia (Surgical Case Presentation)
Benign Prostatic Hyperplasia (Surgical Case Presentation)Benign Prostatic Hyperplasia (Surgical Case Presentation)
Benign Prostatic Hyperplasia (Surgical Case Presentation)
 
Investigations in the case of abdominal pain
Investigations in the case of abdominal painInvestigations in the case of abdominal pain
Investigations in the case of abdominal pain
 

Destaque

Acute abdomen a practical approach
Acute abdomen   a practical approachAcute abdomen   a practical approach
Acute abdomen a practical approachDR Laith
 
Diagnosis And Management Of Acute Abdominal Pain
Diagnosis And Management Of Acute Abdominal PainDiagnosis And Management Of Acute Abdominal Pain
Diagnosis And Management Of Acute Abdominal PainDimitri Raptis
 
respiratory system
respiratory systemrespiratory system
respiratory systemJames Iwu
 
KPSS Güncel Bilgiler 2016 | KPSS Genel Kültür | www.kpssgenelkultur.net
KPSS Güncel Bilgiler 2016 | KPSS Genel Kültür | www.kpssgenelkultur.netKPSS Güncel Bilgiler 2016 | KPSS Genel Kültür | www.kpssgenelkultur.net
KPSS Güncel Bilgiler 2016 | KPSS Genel Kültür | www.kpssgenelkultur.netKpss Genel Kültür
 
Care of aging_skin_fall 2013 abridged
Care of  aging_skin_fall 2013 abridgedCare of  aging_skin_fall 2013 abridged
Care of aging_skin_fall 2013 abridgedShepard Joy
 
History taking format for gyne
History taking format for gyneHistory taking format for gyne
History taking format for gyneBibëk Bhandari
 
The Acute Surgical Abdomen
The Acute Surgical AbdomenThe Acute Surgical Abdomen
The Acute Surgical AbdomenSamuel Gay
 
Urothelial ca urinary markers
Urothelial ca urinary markersUrothelial ca urinary markers
Urothelial ca urinary markersGAURAV NAHAR
 
Causes of abdominal pain
Causes of abdominal painCauses of abdominal pain
Causes of abdominal painAbino David
 
Percutaneous Nephrolithotomy
Percutaneous NephrolithotomyPercutaneous Nephrolithotomy
Percutaneous NephrolithotomySaba Khan
 
Antenatal assessment of fetal well being
Antenatal assessment of fetal well beingAntenatal assessment of fetal well being
Antenatal assessment of fetal well beingAisha Nazeer
 
clinical course" Acute abdomen "
clinical course" Acute abdomen "clinical course" Acute abdomen "
clinical course" Acute abdomen "amin mohamed
 
Percutaneous nephrolithotomy surgery india
Percutaneous nephrolithotomy surgery indiaPercutaneous nephrolithotomy surgery india
Percutaneous nephrolithotomy surgery indiaPankaj Nagpal
 

Destaque (20)

Acute Abdomen
Acute AbdomenAcute Abdomen
Acute Abdomen
 
Acute abdomen a practical approach
Acute abdomen   a practical approachAcute abdomen   a practical approach
Acute abdomen a practical approach
 
Acute Abdominal Pain
Acute Abdominal PainAcute Abdominal Pain
Acute Abdominal Pain
 
Diagnosis And Management Of Acute Abdominal Pain
Diagnosis And Management Of Acute Abdominal PainDiagnosis And Management Of Acute Abdominal Pain
Diagnosis And Management Of Acute Abdominal Pain
 
Acute Abdomine
Acute AbdomineAcute Abdomine
Acute Abdomine
 
respiratory system
respiratory systemrespiratory system
respiratory system
 
KPSS Güncel Bilgiler 2016 | KPSS Genel Kültür | www.kpssgenelkultur.net
KPSS Güncel Bilgiler 2016 | KPSS Genel Kültür | www.kpssgenelkultur.netKPSS Güncel Bilgiler 2016 | KPSS Genel Kültür | www.kpssgenelkultur.net
KPSS Güncel Bilgiler 2016 | KPSS Genel Kültür | www.kpssgenelkultur.net
 
Care of aging_skin_fall 2013 abridged
Care of  aging_skin_fall 2013 abridgedCare of  aging_skin_fall 2013 abridged
Care of aging_skin_fall 2013 abridged
 
History taking format for gyne
History taking format for gyneHistory taking format for gyne
History taking format for gyne
 
The Acute Surgical Abdomen
The Acute Surgical AbdomenThe Acute Surgical Abdomen
The Acute Surgical Abdomen
 
Abdominal Exam
Abdominal ExamAbdominal Exam
Abdominal Exam
 
Pancreas
PancreasPancreas
Pancreas
 
Abdomen exam
Abdomen examAbdomen exam
Abdomen exam
 
Urothelial ca urinary markers
Urothelial ca urinary markersUrothelial ca urinary markers
Urothelial ca urinary markers
 
Causes of abdominal pain
Causes of abdominal painCauses of abdominal pain
Causes of abdominal pain
 
Percutaneous Nephrolithotomy
Percutaneous NephrolithotomyPercutaneous Nephrolithotomy
Percutaneous Nephrolithotomy
 
Antenatal assessment of fetal well being
Antenatal assessment of fetal well beingAntenatal assessment of fetal well being
Antenatal assessment of fetal well being
 
COMPLICATIONS OF PCNL
COMPLICATIONS OF PCNLCOMPLICATIONS OF PCNL
COMPLICATIONS OF PCNL
 
clinical course" Acute abdomen "
clinical course" Acute abdomen "clinical course" Acute abdomen "
clinical course" Acute abdomen "
 
Percutaneous nephrolithotomy surgery india
Percutaneous nephrolithotomy surgery indiaPercutaneous nephrolithotomy surgery india
Percutaneous nephrolithotomy surgery india
 

Semelhante a Acute pain abdomen , clinical examination and reaching for a diagnosis

Acute abdomen the lect .ppt
Acute abdomen the lect .pptAcute abdomen the lect .ppt
Acute abdomen the lect .pptHamedRashad1
 
Acute abdomen – general principles and approach in ED
Acute abdomen – general principles and approach in ED Acute abdomen – general principles and approach in ED
Acute abdomen – general principles and approach in ED DaimaButt1
 
Abdominal examination.pptx
Abdominal examination.pptxAbdominal examination.pptx
Abdominal examination.pptxtsegawbiyazin
 
Health and Physical Assessment
Health and Physical AssessmentHealth and Physical Assessment
Health and Physical AssessmentMelissa Hinnawi
 
L20-Acute Abdomen.pdf
L20-Acute Abdomen.pdfL20-Acute Abdomen.pdf
L20-Acute Abdomen.pdfssusera03368
 
Acute abdominal pain
Acute abdominal painAcute abdominal pain
Acute abdominal painLm Huq
 
Acute abdomen approach to managment-hazem
Acute abdomen approach to managment-hazemAcute abdomen approach to managment-hazem
Acute abdomen approach to managment-hazemmohamedhazemelfoll
 
Abdominal pain and pregnancy
Abdominal  pain and pregnancyAbdominal  pain and pregnancy
Abdominal pain and pregnancydrmcbansal
 
The acute abdomen seminar
The acute abdomen seminarThe acute abdomen seminar
The acute abdomen seminarDrHarsh Saxena
 
Abdominal pain in pregnancy
Abdominal pain in pregnancyAbdominal pain in pregnancy
Abdominal pain in pregnancyAbdu Shumakhi
 
GASTROINTESTINAL SYSTEM EXAMINATION.pptx
GASTROINTESTINAL SYSTEM EXAMINATION.pptxGASTROINTESTINAL SYSTEM EXAMINATION.pptx
GASTROINTESTINAL SYSTEM EXAMINATION.pptxAtthiNaturopathyandY
 
History and examination of acute abdomen by dr fahad akhtar
History and examination of acute  abdomen by dr fahad akhtarHistory and examination of acute  abdomen by dr fahad akhtar
History and examination of acute abdomen by dr fahad akhtarFahad Akhtar
 

Semelhante a Acute pain abdomen , clinical examination and reaching for a diagnosis (20)

Acute abdomen the lect .ppt
Acute abdomen the lect .pptAcute abdomen the lect .ppt
Acute abdomen the lect .ppt
 
Acute abdomen – general principles and approach in ED
Acute abdomen – general principles and approach in ED Acute abdomen – general principles and approach in ED
Acute abdomen – general principles and approach in ED
 
Abdominal examination.pptx
Abdominal examination.pptxAbdominal examination.pptx
Abdominal examination.pptx
 
Health and Physical Assessment
Health and Physical AssessmentHealth and Physical Assessment
Health and Physical Assessment
 
Gerd, gastritis
Gerd, gastritisGerd, gastritis
Gerd, gastritis
 
L20-Acute Abdomen.pdf
L20-Acute Abdomen.pdfL20-Acute Abdomen.pdf
L20-Acute Abdomen.pdf
 
4-acute-abdomen.ppt
4-acute-abdomen.ppt4-acute-abdomen.ppt
4-acute-abdomen.ppt
 
Acute abdominal pain
Acute abdominal painAcute abdominal pain
Acute abdominal pain
 
abdominal assessment
abdominal assessmentabdominal assessment
abdominal assessment
 
Acute abdomen approach to managment-hazem
Acute abdomen approach to managment-hazemAcute abdomen approach to managment-hazem
Acute abdomen approach to managment-hazem
 
Abdominal pain and pregnancy
Abdominal  pain and pregnancyAbdominal  pain and pregnancy
Abdominal pain and pregnancy
 
Abdominal truma 2007
Abdominal truma 2007Abdominal truma 2007
Abdominal truma 2007
 
Approach to abdominal pain
Approach to abdominal painApproach to abdominal pain
Approach to abdominal pain
 
The acute abdomen seminar
The acute abdomen seminarThe acute abdomen seminar
The acute abdomen seminar
 
Abdominal pain in pregnancy
Abdominal pain in pregnancyAbdominal pain in pregnancy
Abdominal pain in pregnancy
 
Abdominal Exam.ppt
Abdominal Exam.pptAbdominal Exam.ppt
Abdominal Exam.ppt
 
Acute Abdomen .pptx
Acute Abdomen .pptxAcute Abdomen .pptx
Acute Abdomen .pptx
 
GASTROINTESTINAL SYSTEM EXAMINATION.pptx
GASTROINTESTINAL SYSTEM EXAMINATION.pptxGASTROINTESTINAL SYSTEM EXAMINATION.pptx
GASTROINTESTINAL SYSTEM EXAMINATION.pptx
 
History and examination of acute abdomen by dr fahad akhtar
History and examination of acute  abdomen by dr fahad akhtarHistory and examination of acute  abdomen by dr fahad akhtar
History and examination of acute abdomen by dr fahad akhtar
 
Inflammatory Bowel Diseases
Inflammatory Bowel DiseasesInflammatory Bowel Diseases
Inflammatory Bowel Diseases
 

Mais de SHANTI MEMORIAL HOSPITAL PVT LTD

THE DANGEROUS TRIAD - OBESITY, DIABETES & HYPERTENSION - IS SURGERY THE SINGL...
THE DANGEROUS TRIAD - OBESITY, DIABETES & HYPERTENSION - IS SURGERY THE SINGL...THE DANGEROUS TRIAD - OBESITY, DIABETES & HYPERTENSION - IS SURGERY THE SINGL...
THE DANGEROUS TRIAD - OBESITY, DIABETES & HYPERTENSION - IS SURGERY THE SINGL...SHANTI MEMORIAL HOSPITAL PVT LTD
 

Mais de SHANTI MEMORIAL HOSPITAL PVT LTD (20)

Patient first Culture
Patient first CulturePatient first Culture
Patient first Culture
 
Painless haematuria for P.G Students
Painless haematuria for P.G Students Painless haematuria for P.G Students
Painless haematuria for P.G Students
 
CBD Stones Technical Challenges
CBD  Stones   Technical ChallengesCBD  Stones   Technical Challenges
CBD Stones Technical Challenges
 
Abcd of lapchole
Abcd of lapchole     Abcd of lapchole
Abcd of lapchole
 
Aging and how to with it
Aging and how to with itAging and how to with it
Aging and how to with it
 
ENDOVISION IN LAPAROSCOPY PPT. DR SREEJOY PATNAIK
ENDOVISION IN LAPAROSCOPY PPT.  DR SREEJOY PATNAIKENDOVISION IN LAPAROSCOPY PPT.  DR SREEJOY PATNAIK
ENDOVISION IN LAPAROSCOPY PPT. DR SREEJOY PATNAIK
 
CONSTIPATION PPT.DR SREEJOY PATNAIK
CONSTIPATION  PPT.DR SREEJOY PATNAIKCONSTIPATION  PPT.DR SREEJOY PATNAIK
CONSTIPATION PPT.DR SREEJOY PATNAIK
 
LAP CBD ppt
LAP CBD  ppt LAP CBD  ppt
LAP CBD ppt
 
Ideal Suture for Surgeons
Ideal Suture for SurgeonsIdeal Suture for Surgeons
Ideal Suture for Surgeons
 
Why robotic bariatric SP
Why robotic bariatric SPWhy robotic bariatric SP
Why robotic bariatric SP
 
Video editing SP
Video editing SPVideo editing SP
Video editing SP
 
Acute pancreatitis SP
Acute pancreatitis SPAcute pancreatitis SP
Acute pancreatitis SP
 
LAPAROSCOPIC UROLOGICAL SURGERY
LAPAROSCOPIC UROLOGICAL SURGERYLAPAROSCOPIC UROLOGICAL SURGERY
LAPAROSCOPIC UROLOGICAL SURGERY
 
Burnout in surgeons
Burnout in surgeonsBurnout in surgeons
Burnout in surgeons
 
WOUND DEHISCENCE
WOUND DEHISCENCEWOUND DEHISCENCE
WOUND DEHISCENCE
 
LAPAROSCOPIC UROLOGY PPT. DR SREEJOY PATNAIK
LAPAROSCOPIC UROLOGY PPT. DR SREEJOY PATNAIKLAPAROSCOPIC UROLOGY PPT. DR SREEJOY PATNAIK
LAPAROSCOPIC UROLOGY PPT. DR SREEJOY PATNAIK
 
THE DANGEROUS TRIAD - OBESITY, DIABETES & HYPERTENSION - IS SURGERY THE SINGL...
THE DANGEROUS TRIAD - OBESITY, DIABETES & HYPERTENSION - IS SURGERY THE SINGL...THE DANGEROUS TRIAD - OBESITY, DIABETES & HYPERTENSION - IS SURGERY THE SINGL...
THE DANGEROUS TRIAD - OBESITY, DIABETES & HYPERTENSION - IS SURGERY THE SINGL...
 
PATHO PHYSIOLOGY OF BARIATRIC SURGERY
PATHO PHYSIOLOGY OF BARIATRIC SURGERYPATHO PHYSIOLOGY OF BARIATRIC SURGERY
PATHO PHYSIOLOGY OF BARIATRIC SURGERY
 
PATHOPHYSIOLOGY OF BARIATRIC SURGERY
PATHOPHYSIOLOGY OF BARIATRIC SURGERYPATHOPHYSIOLOGY OF BARIATRIC SURGERY
PATHOPHYSIOLOGY OF BARIATRIC SURGERY
 
LAPAROSCOPIC SURGERY- PAST, PRESENT AND FUTURE
 LAPAROSCOPIC SURGERY- PAST, PRESENT AND FUTURE LAPAROSCOPIC SURGERY- PAST, PRESENT AND FUTURE
LAPAROSCOPIC SURGERY- PAST, PRESENT AND FUTURE
 

Último

ANG SEKTOR NG agrikultura.pptx QUARTER 4
ANG SEKTOR NG agrikultura.pptx QUARTER 4ANG SEKTOR NG agrikultura.pptx QUARTER 4
ANG SEKTOR NG agrikultura.pptx QUARTER 4MiaBumagat1
 
USPS® Forced Meter Migration - How to Know if Your Postage Meter Will Soon be...
USPS® Forced Meter Migration - How to Know if Your Postage Meter Will Soon be...USPS® Forced Meter Migration - How to Know if Your Postage Meter Will Soon be...
USPS® Forced Meter Migration - How to Know if Your Postage Meter Will Soon be...Postal Advocate Inc.
 
Virtual-Orientation-on-the-Administration-of-NATG12-NATG6-and-ELLNA.pdf
Virtual-Orientation-on-the-Administration-of-NATG12-NATG6-and-ELLNA.pdfVirtual-Orientation-on-the-Administration-of-NATG12-NATG6-and-ELLNA.pdf
Virtual-Orientation-on-the-Administration-of-NATG12-NATG6-and-ELLNA.pdfErwinPantujan2
 
MULTIDISCIPLINRY NATURE OF THE ENVIRONMENTAL STUDIES.pptx
MULTIDISCIPLINRY NATURE OF THE ENVIRONMENTAL STUDIES.pptxMULTIDISCIPLINRY NATURE OF THE ENVIRONMENTAL STUDIES.pptx
MULTIDISCIPLINRY NATURE OF THE ENVIRONMENTAL STUDIES.pptxAnupkumar Sharma
 
Keynote by Prof. Wurzer at Nordex about IP-design
Keynote by Prof. Wurzer at Nordex about IP-designKeynote by Prof. Wurzer at Nordex about IP-design
Keynote by Prof. Wurzer at Nordex about IP-designMIPLM
 
How to Add Barcode on PDF Report in Odoo 17
How to Add Barcode on PDF Report in Odoo 17How to Add Barcode on PDF Report in Odoo 17
How to Add Barcode on PDF Report in Odoo 17Celine George
 
Field Attribute Index Feature in Odoo 17
Field Attribute Index Feature in Odoo 17Field Attribute Index Feature in Odoo 17
Field Attribute Index Feature in Odoo 17Celine George
 
GRADE 4 - SUMMATIVE TEST QUARTER 4 ALL SUBJECTS
GRADE 4 - SUMMATIVE TEST QUARTER 4 ALL SUBJECTSGRADE 4 - SUMMATIVE TEST QUARTER 4 ALL SUBJECTS
GRADE 4 - SUMMATIVE TEST QUARTER 4 ALL SUBJECTSJoshuaGantuangco2
 
ISYU TUNGKOL SA SEKSWLADIDA (ISSUE ABOUT SEXUALITY
ISYU TUNGKOL SA SEKSWLADIDA (ISSUE ABOUT SEXUALITYISYU TUNGKOL SA SEKSWLADIDA (ISSUE ABOUT SEXUALITY
ISYU TUNGKOL SA SEKSWLADIDA (ISSUE ABOUT SEXUALITYKayeClaireEstoconing
 
HỌC TỐT TIẾNG ANH 11 THEO CHƯƠNG TRÌNH GLOBAL SUCCESS ĐÁP ÁN CHI TIẾT - CẢ NĂ...
HỌC TỐT TIẾNG ANH 11 THEO CHƯƠNG TRÌNH GLOBAL SUCCESS ĐÁP ÁN CHI TIẾT - CẢ NĂ...HỌC TỐT TIẾNG ANH 11 THEO CHƯƠNG TRÌNH GLOBAL SUCCESS ĐÁP ÁN CHI TIẾT - CẢ NĂ...
HỌC TỐT TIẾNG ANH 11 THEO CHƯƠNG TRÌNH GLOBAL SUCCESS ĐÁP ÁN CHI TIẾT - CẢ NĂ...Nguyen Thanh Tu Collection
 
4.16.24 21st Century Movements for Black Lives.pptx
4.16.24 21st Century Movements for Black Lives.pptx4.16.24 21st Century Movements for Black Lives.pptx
4.16.24 21st Century Movements for Black Lives.pptxmary850239
 
Judging the Relevance and worth of ideas part 2.pptx
Judging the Relevance  and worth of ideas part 2.pptxJudging the Relevance  and worth of ideas part 2.pptx
Judging the Relevance and worth of ideas part 2.pptxSherlyMaeNeri
 
INTRODUCTION TO CATHOLIC CHRISTOLOGY.pptx
INTRODUCTION TO CATHOLIC CHRISTOLOGY.pptxINTRODUCTION TO CATHOLIC CHRISTOLOGY.pptx
INTRODUCTION TO CATHOLIC CHRISTOLOGY.pptxHumphrey A Beña
 
What is Model Inheritance in Odoo 17 ERP
What is Model Inheritance in Odoo 17 ERPWhat is Model Inheritance in Odoo 17 ERP
What is Model Inheritance in Odoo 17 ERPCeline George
 
ACC 2024 Chronicles. Cardiology. Exam.pdf
ACC 2024 Chronicles. Cardiology. Exam.pdfACC 2024 Chronicles. Cardiology. Exam.pdf
ACC 2024 Chronicles. Cardiology. Exam.pdfSpandanaRallapalli
 
AMERICAN LANGUAGE HUB_Level2_Student'sBook_Answerkey.pdf
AMERICAN LANGUAGE HUB_Level2_Student'sBook_Answerkey.pdfAMERICAN LANGUAGE HUB_Level2_Student'sBook_Answerkey.pdf
AMERICAN LANGUAGE HUB_Level2_Student'sBook_Answerkey.pdfphamnguyenenglishnb
 
Concurrency Control in Database Management system
Concurrency Control in Database Management systemConcurrency Control in Database Management system
Concurrency Control in Database Management systemChristalin Nelson
 
Visit to a blind student's school🧑‍🦯🧑‍🦯(community medicine)
Visit to a blind student's school🧑‍🦯🧑‍🦯(community medicine)Visit to a blind student's school🧑‍🦯🧑‍🦯(community medicine)
Visit to a blind student's school🧑‍🦯🧑‍🦯(community medicine)lakshayb543
 
Culture Uniformity or Diversity IN SOCIOLOGY.pptx
Culture Uniformity or Diversity IN SOCIOLOGY.pptxCulture Uniformity or Diversity IN SOCIOLOGY.pptx
Culture Uniformity or Diversity IN SOCIOLOGY.pptxPoojaSen20
 

Último (20)

ANG SEKTOR NG agrikultura.pptx QUARTER 4
ANG SEKTOR NG agrikultura.pptx QUARTER 4ANG SEKTOR NG agrikultura.pptx QUARTER 4
ANG SEKTOR NG agrikultura.pptx QUARTER 4
 
USPS® Forced Meter Migration - How to Know if Your Postage Meter Will Soon be...
USPS® Forced Meter Migration - How to Know if Your Postage Meter Will Soon be...USPS® Forced Meter Migration - How to Know if Your Postage Meter Will Soon be...
USPS® Forced Meter Migration - How to Know if Your Postage Meter Will Soon be...
 
Virtual-Orientation-on-the-Administration-of-NATG12-NATG6-and-ELLNA.pdf
Virtual-Orientation-on-the-Administration-of-NATG12-NATG6-and-ELLNA.pdfVirtual-Orientation-on-the-Administration-of-NATG12-NATG6-and-ELLNA.pdf
Virtual-Orientation-on-the-Administration-of-NATG12-NATG6-and-ELLNA.pdf
 
MULTIDISCIPLINRY NATURE OF THE ENVIRONMENTAL STUDIES.pptx
MULTIDISCIPLINRY NATURE OF THE ENVIRONMENTAL STUDIES.pptxMULTIDISCIPLINRY NATURE OF THE ENVIRONMENTAL STUDIES.pptx
MULTIDISCIPLINRY NATURE OF THE ENVIRONMENTAL STUDIES.pptx
 
Keynote by Prof. Wurzer at Nordex about IP-design
Keynote by Prof. Wurzer at Nordex about IP-designKeynote by Prof. Wurzer at Nordex about IP-design
Keynote by Prof. Wurzer at Nordex about IP-design
 
How to Add Barcode on PDF Report in Odoo 17
How to Add Barcode on PDF Report in Odoo 17How to Add Barcode on PDF Report in Odoo 17
How to Add Barcode on PDF Report in Odoo 17
 
Field Attribute Index Feature in Odoo 17
Field Attribute Index Feature in Odoo 17Field Attribute Index Feature in Odoo 17
Field Attribute Index Feature in Odoo 17
 
YOUVE_GOT_EMAIL_PRELIMS_EL_DORADO_2024.pptx
YOUVE_GOT_EMAIL_PRELIMS_EL_DORADO_2024.pptxYOUVE_GOT_EMAIL_PRELIMS_EL_DORADO_2024.pptx
YOUVE_GOT_EMAIL_PRELIMS_EL_DORADO_2024.pptx
 
GRADE 4 - SUMMATIVE TEST QUARTER 4 ALL SUBJECTS
GRADE 4 - SUMMATIVE TEST QUARTER 4 ALL SUBJECTSGRADE 4 - SUMMATIVE TEST QUARTER 4 ALL SUBJECTS
GRADE 4 - SUMMATIVE TEST QUARTER 4 ALL SUBJECTS
 
ISYU TUNGKOL SA SEKSWLADIDA (ISSUE ABOUT SEXUALITY
ISYU TUNGKOL SA SEKSWLADIDA (ISSUE ABOUT SEXUALITYISYU TUNGKOL SA SEKSWLADIDA (ISSUE ABOUT SEXUALITY
ISYU TUNGKOL SA SEKSWLADIDA (ISSUE ABOUT SEXUALITY
 
HỌC TỐT TIẾNG ANH 11 THEO CHƯƠNG TRÌNH GLOBAL SUCCESS ĐÁP ÁN CHI TIẾT - CẢ NĂ...
HỌC TỐT TIẾNG ANH 11 THEO CHƯƠNG TRÌNH GLOBAL SUCCESS ĐÁP ÁN CHI TIẾT - CẢ NĂ...HỌC TỐT TIẾNG ANH 11 THEO CHƯƠNG TRÌNH GLOBAL SUCCESS ĐÁP ÁN CHI TIẾT - CẢ NĂ...
HỌC TỐT TIẾNG ANH 11 THEO CHƯƠNG TRÌNH GLOBAL SUCCESS ĐÁP ÁN CHI TIẾT - CẢ NĂ...
 
4.16.24 21st Century Movements for Black Lives.pptx
4.16.24 21st Century Movements for Black Lives.pptx4.16.24 21st Century Movements for Black Lives.pptx
4.16.24 21st Century Movements for Black Lives.pptx
 
Judging the Relevance and worth of ideas part 2.pptx
Judging the Relevance  and worth of ideas part 2.pptxJudging the Relevance  and worth of ideas part 2.pptx
Judging the Relevance and worth of ideas part 2.pptx
 
INTRODUCTION TO CATHOLIC CHRISTOLOGY.pptx
INTRODUCTION TO CATHOLIC CHRISTOLOGY.pptxINTRODUCTION TO CATHOLIC CHRISTOLOGY.pptx
INTRODUCTION TO CATHOLIC CHRISTOLOGY.pptx
 
What is Model Inheritance in Odoo 17 ERP
What is Model Inheritance in Odoo 17 ERPWhat is Model Inheritance in Odoo 17 ERP
What is Model Inheritance in Odoo 17 ERP
 
ACC 2024 Chronicles. Cardiology. Exam.pdf
ACC 2024 Chronicles. Cardiology. Exam.pdfACC 2024 Chronicles. Cardiology. Exam.pdf
ACC 2024 Chronicles. Cardiology. Exam.pdf
 
AMERICAN LANGUAGE HUB_Level2_Student'sBook_Answerkey.pdf
AMERICAN LANGUAGE HUB_Level2_Student'sBook_Answerkey.pdfAMERICAN LANGUAGE HUB_Level2_Student'sBook_Answerkey.pdf
AMERICAN LANGUAGE HUB_Level2_Student'sBook_Answerkey.pdf
 
Concurrency Control in Database Management system
Concurrency Control in Database Management systemConcurrency Control in Database Management system
Concurrency Control in Database Management system
 
Visit to a blind student's school🧑‍🦯🧑‍🦯(community medicine)
Visit to a blind student's school🧑‍🦯🧑‍🦯(community medicine)Visit to a blind student's school🧑‍🦯🧑‍🦯(community medicine)
Visit to a blind student's school🧑‍🦯🧑‍🦯(community medicine)
 
Culture Uniformity or Diversity IN SOCIOLOGY.pptx
Culture Uniformity or Diversity IN SOCIOLOGY.pptxCulture Uniformity or Diversity IN SOCIOLOGY.pptx
Culture Uniformity or Diversity IN SOCIOLOGY.pptx
 

Acute pain abdomen , clinical examination and reaching for a diagnosis

  • 1. EXAMINATION OFEXAMINATION OF THE ABDOMENTHE ABDOMEN Dr.Sreejoy PatnaikDr.Sreejoy Patnaik
  • 2. ACUTE ABDOMENACUTE ABDOMEN One of the most common causes for hospitalizationOne of the most common causes for hospitalization Meaning = acute abdominal symptoms which leadMeaning = acute abdominal symptoms which lead patients to ER , excluding obvious abdominalpatients to ER , excluding obvious abdominal injuriesinjuries May or may not require immediate operationsMay or may not require immediate operations
  • 3. Acute Abdomen ObjectivesAcute Abdomen Objectives Definition of acute abdomen.Definition of acute abdomen. To be able to distinguish between a medical orTo be able to distinguish between a medical or surgical abdomensurgical abdomen To be able to obtain a history to facilitate theTo be able to obtain a history to facilitate the diagnosis:diagnosis: Immediate Management of Life threateningImmediate Management of Life threatening problems: perform a brief examination, identifyproblems: perform a brief examination, identify candidates for urgent surgery.candidates for urgent surgery. Further evaluation of patient with acute abdominalFurther evaluation of patient with acute abdominal pain: H & P / lab. INV./ X-Rays/ special studies.pain: H & P / lab. INV./ X-Rays/ special studies.
  • 4. Acute Abdomen objectivesAcute Abdomen objectives Identify and describe the common localized abdominalIdentify and describe the common localized abdominal massesmasses - umbilical hernia, incisional hernia, epigastric hernia,- umbilical hernia, incisional hernia, epigastric hernia, diastases recti, and lipoma.diastases recti, and lipoma. Describe normal and abnormal-Describe normal and abnormal- - Bowel sounds ( exaggerated)- Bowel sounds ( exaggerated) - Bruits, Venous hum, Friction rubs- Bruits, Venous hum, Friction rubs
  • 5. Acute AbdomenAcute Abdomen objectivesobjectives Identify the areas commonly auscultated forIdentify the areas commonly auscultated for BRUITS. i.e., renal artery stenosisBRUITS. i.e., renal artery stenosis Describe the significance of the abdomen in ALLDescribe the significance of the abdomen in ALL FOUR QUADRANTSFOUR QUADRANTS Describe the common abnormalities that can causeDescribe the common abnormalities that can cause IRREGULAR PERCUSSION NOTES of theIRREGULAR PERCUSSION NOTES of the abdomenabdomen -ovarian tumor, pregnant uterus or GI obstruction-ovarian tumor, pregnant uterus or GI obstruction
  • 6. Acute Abdomen objectivesAcute Abdomen objectives Define and describe the changes associated with lightDefine and describe the changes associated with light and deep palpationand deep palpation To assess any degree of tenderness, i.e. reboundTo assess any degree of tenderness, i.e. rebound tenderness, guardingtenderness, guarding =peritoneal irritation or distended viscous.=peritoneal irritation or distended viscous.
  • 7. Acute abdomenAcute abdomen objectivesobjectives Recognize and perform the special techniques inRecognize and perform the special techniques in examining the abdomen for specific findings:examining the abdomen for specific findings: (1) rebound tenderness(1) rebound tenderness (2) shifting dullness in ascites(2) shifting dullness in ascites (3) Fluid wave in ascites(3) Fluid wave in ascites (4) hooking technique for palpating the liver(4) hooking technique for palpating the liver (5) Murphy's sign for acute cholecystitis(5) Murphy's sign for acute cholecystitis (6) ballottement(6) ballottement (7) the techniques of assessing possible appendicitis(7) the techniques of assessing possible appendicitis
  • 8. Acute AbdomenAcute Abdomen Differential Dx. of Acute Abdominal Pain -Differential Dx. of Acute Abdominal Pain - Anatomic correlation of abdominal painAnatomic correlation of abdominal pain Types of abdominal painTypes of abdominal pain Causes of abdominal pain by quadrantsCauses of abdominal pain by quadrants Parietal pain, visceral pain and referred pain.Parietal pain, visceral pain and referred pain.
  • 9. What does the pain feel likeWhat does the pain feel like ?? Sharp Pain : Biliary ColicSharp Pain : Biliary Colic Tearing Pain : Aortic DissectionTearing Pain : Aortic Dissection PainPain Penetrating Pain : PancreatitisPenetrating Pain : Pancreatitis
  • 10. Acute AbdomenAcute Abdomen H&P: Obtain a complete HxH&P: Obtain a complete Hx Vital signs –Vital signs – Blood pressure ( standing or sitting position)Blood pressure ( standing or sitting position) Pulse & asses peripheral perfusionPulse & asses peripheral perfusion AlertnessAlertness Temperature of skin and extremitiesTemperature of skin and extremities Immediate management of life threateningImmediate management of life threatening Problems: bleeding, shock, hypotensionProblems: bleeding, shock, hypotension
  • 11. Acute AbdomenAcute Abdomen Location of Problem: Chest, abdomen (upper,Location of Problem: Chest, abdomen (upper, middle, lower/ sides)middle, lower/ sides) Time of Onset: Date, time…?Time of Onset: Date, time…? Type of Onset: How: Sudden? Gradual?Type of Onset: How: Sudden? Gradual? Original Source: Triggers, what were youOriginal Source: Triggers, what were you doing? (setting at time of occurrence)doing? (setting at time of occurrence) Severity: Interfere with ADL’S? (Activity ofSeverity: Interfere with ADL’S? (Activity of daily living)daily living) Time Relationship: How often, when?Time Relationship: How often, when? Duration: How long an episode?Duration: How long an episode?
  • 12. Acute AbdomenAcute Abdomen Course: Getting better, worse?Course: Getting better, worse? Association: Any other manifestation?Association: Any other manifestation? Source of Relief: Changes inSource of Relief: Changes in medication, diet? What makes itmedication, diet? What makes it better?better? Source of Aggravation: What makes itSource of Aggravation: What makes it worse?worse? Relevant Data & Pertinent NegativesRelevant Data & Pertinent Negatives
  • 13. Acute AbdomenAcute Abdomen Gynecological Hx for females: last menstrualGynecological Hx for females: last menstrual period, pregnancies, STD’s.period, pregnancies, STD’s. Associated symptoms: nausea, anorexia,Associated symptoms: nausea, anorexia, vomiting, change in bowel habits.vomiting, change in bowel habits.
  • 14. Acute AbdomenAcute Abdomen Dyspnea, SOB, pain, wheezing, crackles, orthopnea,Dyspnea, SOB, pain, wheezing, crackles, orthopnea, (?) Pillows, cough, sputum, emphysema, bronchitis,(?) Pillows, cough, sputum, emphysema, bronchitis, asthma, URL, chest x-rayasthma, URL, chest x-ray Changes in:Changes in: appetite, weight, N/V. abdominal pain, (?) Diet,appetite, weight, N/V. abdominal pain, (?) Diet, digestion, tastes, bowel habits/ stool.digestion, tastes, bowel habits/ stool. Urine: color, polyuria, oliguria, nocturia, dysuria,Urine: color, polyuria, oliguria, nocturia, dysuria, frequency, urgency, stones…frequency, urgency, stones…
  • 15. Acute AbdomenAcute Abdomen Mental statusMental status Abdominal examination (gently) looking for signs ofAbdominal examination (gently) looking for signs of acute abdomen.acute abdomen. Identify and describe the common localizedIdentify and describe the common localized abdominal massesabdominal masses Pelvic(gynecological) exam for females and rectalPelvic(gynecological) exam for females and rectal exam for both male and femaleexam for both male and female (gross blood, asses sphincter tone, and any other(gross blood, asses sphincter tone, and any other evidence of trauma).evidence of trauma). Check for blood in stools: UC, diverticular ds orCheck for blood in stools: UC, diverticular ds or diverticulitis, haemorroidsdiverticulitis, haemorroids
  • 16. PARTS OF THE GUTPARTS OF THE GUT Stomach to 2ndpart of duodenum, includingStomach to 2ndpart of duodenum, including liver, biliary trees, pancreas, and spleen areliver, biliary trees, pancreas, and spleen are derived from FOREGUTderived from FOREGUT 33rdrd and 4thpart of duodenum, jejunum, ileum,and 4thpart of duodenum, jejunum, ileum, appendix, ascending colon to proximal 2/3 ofappendix, ascending colon to proximal 2/3 of transverse colon are derived from MIDGUTtransverse colon are derived from MIDGUT Distal 1/3 of transverse colon to anal canalDistal 1/3 of transverse colon to anal canal above dentate line are derived fromabove dentate line are derived from HINDGUTHINDGUT
  • 17. Pathophysiology of Abdominal painPathophysiology of Abdominal pain Visceral pain from organs derived from FOREGUTVisceral pain from organs derived from FOREGUT and MIDGUT is at midline and above or around theand MIDGUT is at midline and above or around the umbilicus.umbilicus. Visceral pain from organs derived from HINDGUT isVisceral pain from organs derived from HINDGUT is at midline and below the umbilicusat midline and below the umbilicus..
  • 18. ANATOMIC ESSENTIALSANATOMIC ESSENTIALS Abdominal pain isAbdominal pain is typically derived fromtypically derived from one or more threeone or more three distinct pain pathwaysdistinct pain pathways 1. Visceral,1. Visceral, 2.Parietal (Somatic )2.Parietal (Somatic ) 3.Referred3.Referred
  • 19. Peritoneum innervationsPeritoneum innervations VISCERAL PATHWAYVISCERAL PATHWAY Sympathetic and parasympathetic nerveSympathetic and parasympathetic nerve innervations(C fibers)innervations(C fibers) CharacterCharacter -dull or cramping pain-dull or cramping pain -insidious-insidious -sensitive to distension, ischemia, squeezing,-sensitive to distension, ischemia, squeezing, torsiontorsion -insensitive to heat, cutting, or electrical shock-insensitive to heat, cutting, or electrical shock
  • 20. The visceral Abdominal painThe visceral Abdominal pain Visceral pain fibers areVisceral pain fibers are bilateral, unmyelinated andbilateral, unmyelinated and enter the spinal cord atenter the spinal cord at multiple levels .multiple levels . The visceral abdominal pain isThe visceral abdominal pain is usually dull , , poorly localizedusually dull , , poorly localized and experienced in the midline .and experienced in the midline .
  • 21. Visceral Abdominal PainVisceral Abdominal Pain Visceral Abdominal PainVisceral Abdominal Pain is usually caused byis usually caused by distention of hollowdistention of hollow organs or capsularorgans or capsular stretching of solidstretching of solid organs.organs.
  • 22. Visceral Abdominal PainVisceral Abdominal Pain Less commonly , it is caused by ischemia orLess commonly , it is caused by ischemia or inflammation.inflammation. The tissue congestion sensitizes nerve endings ofThe tissue congestion sensitizes nerve endings of visceral pain fibers and lowers the threshold forvisceral pain fibers and lowers the threshold for stimulus.stimulus.
  • 23. Visceral Abdominal PainVisceral Abdominal Pain If the involved organ isIf the involved organ is affected by peristalsis,affected by peristalsis, the pain is oftenthe pain is often described asdescribed as intermittent, cramp, orintermittent, cramp, or colicky in naturecolicky in nature
  • 24. PARIETAL PATHWAYPARIETAL PATHWAY Somatic nerve innervations (A fibers)Somatic nerve innervations (A fibers) Somatic nerve distribution (T7-L2, umbilicus at T12)Somatic nerve distribution (T7-L2, umbilicus at T12) CharacterCharacter -----sharp and exquisite pain-----sharp and exquisite pain Sensitive to mechanical stimuli (stretching, pinprick,Sensitive to mechanical stimuli (stretching, pinprick, pinch), heat, electrical shock, chemical stimulus,pinch), heat, electrical shock, chemical stimulus, infection-inflammationinfection-inflammation
  • 25. Parietal (Somatic) AbdominalParietal (Somatic) Abdominal painpain Results fromResults from ischemia, ,ischemia, , inflammation , orinflammation , or stretching of thestretching of the parietalparietal Peritoneum.Peritoneum.
  • 26. Parietal (Somatic ) AbdominalParietal (Somatic ) Abdominal PainPain Myelinated afferent fibersMyelinated afferent fibers that transmit the painfulthat transmit the painful stimulus to specific dorsalstimulus to specific dorsal root ganglia on the same sideroot ganglia on the same side and dermatomal level at theand dermatomal level at the origin of the painorigin of the pain
  • 27. Parietal (Somatic ) AbdominalParietal (Somatic ) Abdominal PainPain The parietal pain , in contrast to visceral painThe parietal pain , in contrast to visceral pain pain, , often can be localized to the region ofpain, , often can be localized to the region of the painful stimulus.the painful stimulus. • This pain is typically sharp, knife- like andThis pain is typically sharp, knife- like and constant; coughing and moving are likelyconstant; coughing and moving are likely likely to aggravate it.likely to aggravate it.
  • 28. • The classic presentation ofThe classic presentation of appendicitis involves bothappendicitis involves both visceral and parietal pain.visceral and parietal pain. The pain of early presentation isThe pain of early presentation is often periumbilical (visceral )often periumbilical (visceral ) but localizes to the right lowerbut localizes to the right lower quadrant ( RLQ) when thequadrant ( RLQ) when the inflammation extends to theinflammation extends to the peritoneum (parietalperitoneum (parietal).).
  • 29. REFERRED PAINREFERRED PAIN • Is defined as pain felt at aIs defined as pain felt at a distance from the diseaseddistance from the diseased organ .organ . • It results from sharedIt results from shared central pathways forcentral pathways for afferent neurons fromafferent neurons from different locations .different locations .
  • 30. ASSESSMENTASSESSMENT 2 MOST IMPORTANT THINGS in assessment of the2 MOST IMPORTANT THINGS in assessment of the patients are--patients are-- • Careful and precise history taking, andCareful and precise history taking, and • Physical examinationPhysical examination
  • 31. BASIC HISTORY TAKINGBASIC HISTORY TAKING • ONSETONSET • 1. SUDDEN1. SUDDEN • - Perforated PU, Gallstone, UC, Aortic dissection,- Perforated PU, Gallstone, UC, Aortic dissection, Rupture AAA, SMA Embolism, Ruptured ectopicRupture AAA, SMA Embolism, Ruptured ectopic preg., Ruptured corpus lateral or follicular cysts,preg., Ruptured corpus lateral or follicular cysts, Twisted ovarian cystTwisted ovarian cyst • 2. INSIDIOUS2. INSIDIOUS • - Acute Appendicitis, Acute pancreatitis, Intestinal- Acute Appendicitis, Acute pancreatitis, Intestinal obstruction, Acute pyelonephritis, Acute gastritis orobstruction, Acute pyelonephritis, Acute gastritis or gastroenteritisgastroenteritis
  • 32. AGEAGE 1.1. CHILDHOOD:CHILDHOOD: Constipation, Acute appendicitis, Intussusceptions,Constipation, Acute appendicitis, Intussusceptions, Viral enteritisViral enteritis 2.ADULTS::2.ADULTS:: Infecion-inflammation,Female reproductive organsInfecion-inflammation,Female reproductive organs 3.MIDDLE TO OLD AGE :3.MIDDLE TO OLD AGE : Malignancies, Degenerative diseasesMalignancies, Degenerative diseases
  • 33. NATURE OF PAINNATURE OF PAIN COLICKY SHARPSHOOTING,COLICKY SHARPSHOOTING, intermittent, restless, associated withintermittent, restless, associated with vomitingvomiting -is likely from acute obstruction of hollow-is likely from acute obstruction of hollow viscous organs (small bowel, biliary trees,viscous organs (small bowel, biliary trees, ureter,or even appendix).ureter,or even appendix). -SMA occlusion maybe possible-SMA occlusion maybe possible -Acute Gastritis, Gastroenteritis-Acute Gastritis, Gastroenteritis
  • 34. SUDDEN,SHARP & PERSISTENT:SUDDEN,SHARP & PERSISTENT: -Leakage of irritating fluid, i.e. blood from-Leakage of irritating fluid, i.e. blood from Ruptured ectopic preg,Ruptured ectopic preg, -AAA,-AAA, -Corpus luteal or follicular cysts, --Corpus luteal or follicular cysts, - -HaematomaFluid from ovarian cyst,-HaematomaFluid from ovarian cyst, -Perforated PU-Perforated PU SHEARING OR TEARING:SHEARING OR TEARING: -Aortic dissection,-Aortic dissection, -Ruptured AAA-Ruptured AAA
  • 35. • ASSOCIATED SYMPTOMSASSOCIATED SYMPTOMS : Nausea,vomiting, resp.: Nausea,vomiting, resp. symptomssymptoms • BOWEL HABITS :BOWEL HABITS : Diarrhea,constipation,Diarrhea,constipation, mucous bloody stoolmucous bloody stool • GYNECOLOGIC HISTORYGYNECOLOGIC HISTORY :: menstruation,leucorrhea, sexual intercoursemenstruation,leucorrhea, sexual intercourse • CONCOMITANT HISTORYCONCOMITANT HISTORY:: • Underlying diseasesUnderlying diseases • Family HistoryFamily History • Drug usageDrug usage • Substance expSubstance exposureosure
  • 36. ABDOMINALASSESSMENTABDOMINALASSESSMENT LANDMARKSLANDMARKS • 1. Xiphoid Process1. Xiphoid Process • 2. Costal Margin2. Costal Margin • 3. Abdominal Midline3. Abdominal Midline • 4. Umbilicus4. Umbilicus • 5. Rectus Abdominis Muscle5. Rectus Abdominis Muscle • 6.Ant. Sup. Iliac Spine6.Ant. Sup. Iliac Spine • 7. Inguinal Ligament7. Inguinal Ligament (Poupart’s Ligament)(Poupart’s Ligament) • 8. Symphysis Pubis8. Symphysis Pubis
  • 37. Position of the patient and exposure forPosition of the patient and exposure for abdominal examination. Note that theabdominal examination. Note that the genitalia must be exposed.genitalia must be exposed.
  • 38. ASSESSMENT OF THE ABDOMENASSESSMENT OF THE ABDOMEN INSPECTION -- CONTOURINSPECTION -- CONTOUR • TYPES OFTYPES OF ABDOMEN:ABDOMEN: • FlatFlat • rounded or convexrounded or convex • ScaphoidScaphoid • protuberantprotuberant
  • 39. Assessment of the AbdomenAssessment of the Abdomen Inspection -- ContourInspection -- Contour • Flat is normalFlat is normal LARGE CONVEX ABDOMEN -- 7 F’SLARGE CONVEX ABDOMEN -- 7 F’S FatFat FaecesFaeces Fluid (ascites)Fluid (ascites) FoetusFoetus FlatusFlatus Fatal growth (malignancy)Fatal growth (malignancy) Fibroid tumorFibroid tumor
  • 40. ASSESSMENT OF THE ABDOMENASSESSMENT OF THE ABDOMEN INSPECTION -- CONTOURINSPECTION -- CONTOUR • CONCAVE OR SCAPHOID ABDOMENCONCAVE OR SCAPHOID ABDOMEN – Decreased fat depositsDecreased fat deposits – Malnourished stateMalnourished state – Flaccid muscle toneFlaccid muscle tone • CONVEX OR PROTUBERANT ABDOMEN -- 7CONVEX OR PROTUBERANT ABDOMEN -- 7 F’s: fat, fluid (ascites), flatus, faeces, foetus, fatalF’s: fat, fluid (ascites), flatus, faeces, foetus, fatal growth (malignancy), fibroid tumorgrowth (malignancy), fibroid tumor
  • 41. ASSESSMENT OF THE ABDOMENASSESSMENT OF THE ABDOMEN INSPECTION -- SYMMETRYINSPECTION -- SYMMETRY • THE ABDOMEN SHOULD BE SYMMETRICALTHE ABDOMEN SHOULD BE SYMMETRICAL BILATERALBILATERAL • Asymmetry indicatesAsymmetry indicates – tumortumor – cystscysts – bowel obstructionbowel obstruction – enlargement of abdominal organsenlargement of abdominal organs – scoliosisscoliosis
  • 42. ASSESSMENT OF THE ABDOMENASSESSMENT OF THE ABDOMEN • INSPECTION -- RECTUS ABDOMINIS MUSCLEINSPECTION -- RECTUS ABDOMINIS MUSCLE • Normal -- no ridge separating the musclesNormal -- no ridge separating the muscles • When a ridge is present - diastasis recti abdominusWhen a ridge is present - diastasis recti abdominus – marked obesitymarked obesity – past pregnancypast pregnancy – increased intra-abdominal pressureincreased intra-abdominal pressure
  • 43. ASSESSMENT OF THE ABDOMENASSESSMENT OF THE ABDOMEN RESPIRATORY MOVEMENTRESPIRATORY MOVEMENT INSPECTIONINSPECTION • Normally no retractions -- the abdomen rises andNormally no retractions -- the abdomen rises and falls with each respirationfalls with each respiration • Abnormal due to abdominal disordersAbnormal due to abdominal disorders – appendicitis with local peritonitisappendicitis with local peritonitis – pancreatitispancreatitis – biliary colic or ac. cholecystitisbiliary colic or ac. cholecystitis – perforated ulcerperforated ulcer
  • 44. ASSESSMENT OF THE ABDOMENASSESSMENT OF THE ABDOMEN • Masses or nodulesMasses or nodules -- tumors, metastasis, internal malignancy, or-- tumors, metastasis, internal malignancy, or pregnancypregnancy • Visible PeristalsisVisible Peristalsis -- indicative of obstruction-- indicative of obstruction • PulsationPulsation -- aortic aneurysm or may occur in aortic-- aortic aneurysm or may occur in aortic regurgitation and in right ventricular hypertrophyregurgitation and in right ventricular hypertrophy
  • 45. ASSESSMENT OF THE ABDOMENASSESSMENT OF THE ABDOMEN UMBILICUSUMBILICUS • Umbilical HerniaUmbilical Hernia -- protrusion of the umbilicus-- protrusion of the umbilicus secondary to non closure of the ring permittingsecondary to non closure of the ring permitting intestine or omentum to protrudeintestine or omentum to protrude • Sister Mary Joseph’s noduleSister Mary Joseph’s nodule -- nodule in the-- nodule in the umbilicus secondary to CA intra-abdominalumbilicus secondary to CA intra-abdominal • Intra-abdominal pressureIntra-abdominal pressure -- protrusion from ascites,-- protrusion from ascites, masses, or pregnancymasses, or pregnancy
  • 46. Some commonly used abdominal incisions. The midlineSome commonly used abdominal incisions. The midline and oblique incisions avoid damage to the innervation ofand oblique incisions avoid damage to the innervation of the abdominal musculature and the later developmentthe abdominal musculature and the later development of incisional herniaeof incisional herniae
  • 47. Assessment of the AbdomenAssessment of the Abdomen PALPATIONPALPATION • Please warm your handsPlease warm your hands • To start avoid known tender areasTo start avoid known tender areas • Tell the patient take slow deep breaths throughTell the patient take slow deep breaths through mouthmouth • Begin with gentle pressure then increaseBegin with gentle pressure then increase • If ticklish or with children palpate throughIf ticklish or with children palpate through their hand till ticklishness is gonetheir hand till ticklishness is gone • Avoid quick, short jabsAvoid quick, short jabs • Observe patient’s face for expressions of painObserve patient’s face for expressions of pain
  • 48. Downloaded from: StudentConsult (on 25 March 2012 05:22 PM) © 2005 Elsevier Correct method of palpation. The hand is held flat and relaxed, and 'moulded' to the abdominal wall.
  • 49. Downloaded from: StudentConsult (on 25 March 2012 05:22 PM) © 2005 Elsevier Incorrect method of palpation. The hand is held rigid and mostly not in contact with the abdominal wall.
  • 50. Method of deep palpation in an obese, muscular or poorly relaxed patient.
  • 51. Assessment of the AbdomenAssessment of the Abdomen PALPATION -- LIGHTPALPATION -- LIGHT • Lightly palpate to noteLightly palpate to note – Skin temperatureSkin temperature – TendernessTenderness – Large massesLarge masses
  • 52. Assessment of the AbdomenAssessment of the Abdomen Palpation -- Abdominal Muscle GuardingPalpation -- Abdominal Muscle Guarding • Use both hands -- one on each rectusUse both hands -- one on each rectus • Check for tensing during expirationCheck for tensing during expiration • When positive it is indicative of peritonealWhen positive it is indicative of peritoneal irritation -- peritonitisirritation -- peritonitis
  • 53. Assessment of the AbdomenAssessment of the Abdomen Palpation -- DeepPalpation -- Deep • You can use one or two handed methodYou can use one or two handed method • Two handed method is usually used in obese orTwo handed method is usually used in obese or very muscular individualsvery muscular individuals • Palpate all quadrantsPalpate all quadrants
  • 54. Assessment of the AbdomenAssessment of the Abdomen Palpation -- Fluid WavePalpation -- Fluid Wave • With an assistantWith an assistant placing the ulnarplacing the ulnar surface of their handsurface of their hand firmly in the midlinefirmly in the midline of the patientof the patient • You tap from one sideYou tap from one side to feel the wave on theto feel the wave on the other sideother side • Present withPresent with ascitesascites
  • 55. Assessment of the AbdomenAssessment of the Abdomen PalpationPalpation ---- Liver -- Bimanual MethodLiver -- Bimanual Method • Left hand under patient’sLeft hand under patient’s right flank (11th-12thright flank (11th-12th rib) press upwardrib) press upward • Place right hand at levelPlace right hand at level of dullness -- have patientof dullness -- have patient take a deep breathtake a deep breath • Push in deeplyPush in deeply • note -- size, shape,note -- size, shape, consistency, or massesconsistency, or masses
  • 56. Assessment of the AbdomenAssessment of the Abdomen Palpation -- Liver -- Hook MethodPalpation -- Liver -- Hook Method • Place both hands side byPlace both hands side by side below the level of liverside below the level of liver dullnessdullness • Hook fingers in and up andHook fingers in and up and have the patient take a deephave the patient take a deep breath inbreath in • Note the size, shape,Note the size, shape, consistency, and any massesconsistency, and any masses • I prefer to do this in a sittingI prefer to do this in a sitting positionposition
  • 57. Assessment of the AbdomenAssessment of the Abdomen Palpation -- Liver -- HepatomegalyPalpation -- Liver -- Hepatomegaly Enlarged liverEnlarged liver – congestive heart failurecongestive heart failure – hepatitishepatitis – encephalopathyencephalopathy – cirrhosiscirrhosis – cystcyst – cancercancer
  • 58. Assessment of the AbdomenAssessment of the Abdomen Palpation -- Liver -- Murphy’s SignPalpation -- Liver -- Murphy’s Sign • Palpate the liver margin at the lateral borderPalpate the liver margin at the lateral border of the rectus muscleof the rectus muscle • Have the patient take a deep breathHave the patient take a deep breath • If patient exhibits pain and stops inhaling thisIf patient exhibits pain and stops inhaling this is a positiveis a positive Murphy’s SignMurphy’s Sign present inpresent in CholecystitisCholecystitis
  • 59. Assessment of the AbdomenAssessment of the Abdomen Palpation -- Spleen -- Bimanual TechniquePalpation -- Spleen -- Bimanual Technique • Pull up with left hand andPull up with left hand and push in with right hand onpush in with right hand on inspirationinspiration • Will only be able to feel if 3Will only be able to feel if 3 times normal sizetimes normal size • SplenomegalySplenomegaly – inflammationinflammation – congestive heart failurecongestive heart failure – cancercancer – cirrhosiscirrhosis
  • 60. Assessment of the AbdomenAssessment of the Abdomen Palpation -- Kidney -- Bimanual TechniquePalpation -- Kidney -- Bimanual Technique • Place one hand on thePlace one hand on the costovertebral angle of thecostovertebral angle of the back and the other handback and the other hand just below the costaljust below the costal marginmargin • Increase pressure duringIncrease pressure during inspiration then haveinspiration then have patient hold breathpatient hold breath
  • 61. Assessment of the AbdomenAssessment of the Abdomen Palpation -- KidneyPalpation -- Kidney • The right kidney maybe difficult to distinguishThe right kidney maybe difficult to distinguish from an enlarged liverfrom an enlarged liver • Left kidney enlargement maybe difficult toLeft kidney enlargement maybe difficult to distinguish from an enlarged spleendistinguish from an enlarged spleen • Enlarged palpable kidneys are secondary to:Enlarged palpable kidneys are secondary to: – hydronephrosishydronephrosis – neoplasmsneoplasms – polycystic diseasepolycystic disease
  • 62. Assessment of the AbdomenAssessment of the Abdomen Palpation -- AortaPalpation -- Aorta • Assess the width of theAssess the width of the aorta by placing youraorta by placing your hands on each side ofhands on each side of the aorta just abovethe aorta just above the umbilicusthe umbilicus • Abdominal aorticAbdominal aortic aneurysmaneurysm -- width-- width greater than 4 cm withgreater than 4 cm with lateral pulsationslateral pulsations
  • 63. AssesAssessment of the Abdomensment of the Abdomen Rebound TendernessRebound Tenderness • Apply firm pressure for several seconds to theApply firm pressure for several seconds to the abdomen with hand at right angles and fingersabdomen with hand at right angles and fingers extendedextended • Quickly release the pressureQuickly release the pressure • Test away from site where pain is initially determinedTest away from site where pain is initially determined
  • 64. Assessment of the AbdomenAssessment of the Abdomen Rebound TendernessRebound Tenderness • Pain at site is direct rebound tendernessPain at site is direct rebound tenderness • Pain at another site is referred reboundPain at another site is referred rebound tendernesstenderness • Indicative of peritoneal inflammationIndicative of peritoneal inflammation • If in the RLQ think appendicitisIf in the RLQ think appendicitis (McBurney’s point -- to be discussed later)(McBurney’s point -- to be discussed later)
  • 65. Assessment of the AbdomenAssessment of the Abdomen Rovsing’s SignRovsing’s Sign • Press in the LLQ evenly for 5 secondsPress in the LLQ evenly for 5 seconds and note if patient has pain in RLQ -and note if patient has pain in RLQ - positivepositive • Gas is pushed through the ileocecalGas is pushed through the ileocecal valve thus distending the cecumvalve thus distending the cecum • InIn appendicitisappendicitis pain is notedpain is noted
  • 66. Assessment of the AbdomenAssessment of the Abdomen Cutaneous HypersensitivityCutaneous Hypersensitivity • Either lift the skin or stimulate the skin withEither lift the skin or stimulate the skin with gentle jabbing with a sterile pingentle jabbing with a sterile pin • Indicates a zone of peritoneal irritationIndicates a zone of peritoneal irritation – RLQRLQ -- appendicitis-- appendicitis – Mid Epigastrium --Mid Epigastrium -- peptic ulcerpeptic ulcer
  • 67. Assessment of the AbdomenAssessment of the Abdomen Iliopsoas Muscle TestIliopsoas Muscle Test • Place your hand overPlace your hand over the right thigh andthe right thigh and push downward as thepush downward as the patient is trying topatient is trying to raise the leg, flexingraise the leg, flexing the hipthe hip • Positive RLQ painPositive RLQ pain associated with aassociated with a retrocecal orretrocecal or perforated appendicitisperforated appendicitis
  • 68. Assessment of the AbdomenAssessment of the Abdomen Obturator Muscle TestObturator Muscle Test • Flex the right leg at theFlex the right leg at the hip and knee at a righthip and knee at a right angle then rotate the legangle then rotate the leg internally andinternally and externallyexternally • Pain indicative ofPain indicative of inflammatory processinflammatory process over obturator muscleover obturator muscle – Ruptured appendixRuptured appendix – Pelvic abscessPelvic abscess
  • 69. Assessment of the AbdomenAssessment of the Abdomen BallottmentBallottment • Used to displaceUsed to displace excess fluid in theexcess fluid in the abdominal cavity byabdominal cavity by using stiffened fingersusing stiffened fingers in a jabbing motionin a jabbing motion • Determines a freeDetermines a free floating massfloating mass • If pain is associatedIf pain is associated with an inflammatorywith an inflammatory processprocess
  • 70. Assessment of the AbdomenAssessment of the Abdomen Bladder PalpationBladder Palpation • Using deep palpationUsing deep palpation start at the symphisisstart at the symphisis pubis and palpate uppubis and palpate up • Nodular bladder --Nodular bladder -- malignancymalignancy • Asymmetrical bladderAsymmetrical bladder – tumor in the bladdertumor in the bladder – abdominal tumorabdominal tumor causing compressioncausing compression
  • 71. Assessment of the AbdomenAssessment of the Abdomen Percussion -- GeneralPercussion -- General • Visualize all organs as youVisualize all organs as you percuss all quadrantspercuss all quadrants • Tympanitic-Tympanitic- heard mostlyheard mostly -- hollow organs.-- hollow organs. • DullDull sound- over solidsound- over solid organsorgans • Dull sound in area whereDull sound in area where it should be tympanitic --it should be tympanitic -- mass or tumor, pregnancy,mass or tumor, pregnancy, ascitesascites • Liver dullness- obliteratedLiver dullness- obliterated in perforation.in perforation.
  • 72. Assessment of the AbdomenAssessment of the Abdomen Percussion -- Liver SpanPercussion -- Liver Span • Percuss 7cm upward -- midclavicular line above thePercuss 7cm upward -- midclavicular line above the umbilicus -- typany to dull -- markumbilicus -- typany to dull -- mark • Percuss downward -- midclavicular line -- tympany toPercuss downward -- midclavicular line -- tympany to dull - markdull - mark • Normal - 6-12 cm in males & 10.5 cm in femalesNormal - 6-12 cm in males & 10.5 cm in females
  • 73. Assessment of the AbdomenAssessment of the Abdomen Percussion – Ascites (Shifting Dullness)Percussion – Ascites (Shifting Dullness) • Percuss from above and below from dullness toPercuss from above and below from dullness to tympany while patient on their backtympany while patient on their back • Place patient on left side and percuss fromPlace patient on left side and percuss from dullness to tympanydullness to tympany
  • 74. Assessment of the AbdomenAssessment of the Abdomen Percussion -- AscitesPercussion -- Ascites • The umbilical area percusses dull because theThe umbilical area percusses dull because the ascitic fluid pools in the dependent areaascitic fluid pools in the dependent area • Ascites is present in cirrhosis and other liverAscites is present in cirrhosis and other liver diseasesdiseases
  • 75. Assessment of the AbdomenAssessment of the Abdomen Fist Percussion -- KidneyFist Percussion -- Kidney • Direct fist percussion -- strike the costovertebral angleDirect fist percussion -- strike the costovertebral angle with one fistwith one fist • Indirect fist percussion -- place palm overIndirect fist percussion -- place palm over costovertebral angle and strike with other handcostovertebral angle and strike with other hand • Pain indicative of infllammatory conditionPain indicative of infllammatory condition
  • 76. Assessment of the AbdomenAssessment of the Abdomen Fist Percussion -- LiverFist Percussion -- Liver • Palm down RUQ --Palm down RUQ -- strike with other handstrike with other hand • Tenderness can occurTenderness can occur with:with: – PyelonephritisPyelonephritis – CholecystitisCholecystitis – HepatitisHepatitis
  • 77. Assessment of the AbdomenAssessment of the Abdomen Percussion -- BladderPercussion -- Bladder • Percuss from symphisis pubis (urinePercuss from symphisis pubis (urine filled gives dull sound) - if patientfilled gives dull sound) - if patient unable to empty it is secondary to:unable to empty it is secondary to: – Neurogenic dysfuntionNeurogenic dysfuntion – Benign prostatic hypertrophyBenign prostatic hypertrophy – Post-op casePost-op case – Urethral strictureUrethral stricture – Some medicationsSome medications
  • 78. ASSESSMENT OF THE ABDOMENASSESSMENT OF THE ABDOMEN AUSCULTATIONAUSCULTATION • Use diaphragm of stethoscopeUse diaphragm of stethoscope lightly placedlightly placed • RLQ->RUQ->LUQ->LLQRLQ->RUQ->LUQ->LLQ • Bowel sounds 5-30/minBowel sounds 5-30/min • High-pitched always heardHigh-pitched always heard RLQ-ileo-ceacal areaRLQ-ileo-ceacal area • Borborygmi -- “stomachBorborygmi -- “stomach growling” -- normal,growling” -- normal, hyperactive, gurgling soundhyperactive, gurgling sound
  • 79. BOWEL SOUNDSBOWEL SOUNDS • Bowel Sounds are high pitched sounds & are classifiedBowel Sounds are high pitched sounds & are classified into 4 categoriesinto 4 categories • 1.Normal1.Normal • 2.Hypoactive2.Hypoactive • 3.Absent3.Absent • 4.Hyperactive4.Hyperactive
  • 80. Normal Bowel SoundsNormal Bowel Sounds • Normal Bowel Sounds areNormal Bowel Sounds are high pitched with Clicks andhigh pitched with Clicks and Gargles , which occur every 5Gargles , which occur every 5 to 15 seconds intervalsto 15 seconds intervals
  • 84. ASSESSMENT OF THE ABDOMENASSESSMENT OF THE ABDOMEN AUSCULTATION -- BOWEL SOUNDSAUSCULTATION -- BOWEL SOUNDS • Absent bowel sounds -- no sound for 4-5minutes--Absent bowel sounds -- no sound for 4-5minutes-- Late intestinal obstructionLate intestinal obstruction • Mechanical obstruction -- adhesions, hernias,Mechanical obstruction -- adhesions, hernias, massesmasses • Non-mechanical obstruction -- no intestinalNon-mechanical obstruction -- no intestinal contraction (paralytic ileus) -- physiological,contraction (paralytic ileus) -- physiological, neurogenic, and chemical imbalancesneurogenic, and chemical imbalances
  • 85. Assessment of the AbdomenAssessment of the Abdomen Auscultation -- Bowel SoundsAuscultation -- Bowel Sounds • Hypoactive bowel sounds -- indicatesHypoactive bowel sounds -- indicates decreased motilitydecreased motility – PeritonitisPeritonitis – Non-mechanical obstructionNon-mechanical obstruction – InflammationInflammation – GangreneGangrene – Electrolyte imbalancesElectrolyte imbalances – Intraoperative manipulation of theIntraoperative manipulation of the bowelbowel
  • 86. Assessment of the AbdomenAssessment of the Abdomen Auscultation -- Bowel SoundsAuscultation -- Bowel Sounds • Hyperactive bowel sounds -- increasedHyperactive bowel sounds -- increased motility of the bowelmotility of the bowel – GastroenteritisGastroenteritis – DiarrheaDiarrhea – Laxative useLaxative use – Subsiding ileusSubsiding ileus
  • 87. Assessment of the AbdomenAssessment of the Abdomen Auscultation -- Bowel SoundsAuscultation -- Bowel Sounds • High pitched tinkling hyperactive bowelHigh pitched tinkling hyperactive bowel soundssounds – Caused by powerful peristaltic actionCaused by powerful peristaltic action indicative of partial obstructionindicative of partial obstruction – Abdominal crampingAbdominal cramping
  • 88. Assessment of the AbdomenAssessment of the Abdomen Auscultation -- Venous humAuscultation -- Venous hum • A continuous pulsing or fibrillary soundA continuous pulsing or fibrillary sound • If present in the periumbilical area isIf present in the periumbilical area is secondary to portal vein obstructionsecondary to portal vein obstruction • Portal hypertension caused by cirrhosisPortal hypertension caused by cirrhosis
  • 89. Assessment of the AbdomenAssessment of the Abdomen Auscultation -- Friction RubsAuscultation -- Friction Rubs • Using the diaphragm of the stethoscopeUsing the diaphragm of the stethoscope a sound similar to rubbing sandpapera sound similar to rubbing sandpaper togethertogether • Sound increases with inspirationSound increases with inspiration – TumorsTumors – InflammationInflammation – InfarctionInfarction
  • 90. Assessment of the AbdomenAssessment of the Abdomen Other Examination:Other Examination: • Hernia ExaminationHernia Examination • Pelvic ExaminationPelvic Examination • Rectal examinationRectal examination • Laboratory ExaminationLaboratory Examination • Diagnostic ImagingDiagnostic Imaging
  • 91. Acute AbdomenAcute Abdomen Associated symptoms.Associated symptoms. ECG: inferior wall MI may present withECG: inferior wall MI may present with epigastric or upper abdominal painepigastric or upper abdominal pain with no clear disorder identifiedwith no clear disorder identified Identify conditions for urgent surgery:Identify conditions for urgent surgery: hypotension without GI bleeding,hypotension without GI bleeding, aneurysm, rigid abdomen in a pt withaneurysm, rigid abdomen in a pt with abdominal pain,.abdominal pain,.
  • 92. Acute abdomenAcute abdomen Treat shock.Treat shock. Determine shock severity:Determine shock severity: 1) compensated shock1) compensated shock 2) decompensated shock2) decompensated shock
  • 93. Acute abdomenAcute abdomen • -First Understand the-First Understand the • -Anatomic correlation of abdominal pain-Anatomic correlation of abdominal pain - Types of abdominal pain:- Types of abdominal pain: -Causes of abdominal pain by quadrants,-Causes of abdominal pain by quadrants, -Parietal pain, visceral pain and referred pain.-Parietal pain, visceral pain and referred pain. -Most common surgical Causes of-Most common surgical Causes of RUQ:RUQ: abdominal painabdominal pain • -Perforated duodenal ulcer-Perforated duodenal ulcer • -Cholecystitis-Cholecystitis
  • 94. The Acute AbdomenThe Acute Abdomen Rapid Onset of PainRapid Onset of Pain
  • 95. The Acute AbdomenThe Acute Abdomen Slow Onset of PainSlow Onset of Pain
  • 96. Acute AbdomenAcute Abdomen • Hepatic abscessHepatic abscess • Retrocecal appendicitisRetrocecal appendicitis • Appendicitis in pregnant womanAppendicitis in pregnant woman RLQ:RLQ: • AppendicitisAppendicitis • Cecal diverticulitisCecal diverticulitis • Meckel’s diverticulitisMeckel’s diverticulitis
  • 97. Acute abdomenAcute abdomen LLQ:LLQ: • Splenic ruptureSplenic rupture • Splenic abscessSplenic abscess LUQ:LUQ: • Splenic ruptureSplenic rupture • Splenic abscessSplenic abscess Diffuse pain:Diffuse pain: • Bowel obstructionBowel obstruction • Leaking aneurismLeaking aneurism • MesentericMesenteric • IsquemiaIsquemia Periumbilical:Periumbilical: • Early appendicitisEarly appendicitis • Referred pain fromReferred pain from small bowelsmall bowel
  • 98. Acute abdomenAcute abdomen • NonSurgical causes of abdominal pain:NonSurgical causes of abdominal pain: • RUQ:RUQ: • (RLL) pneumonia(RLL) pneumonia • Biliary colicBiliary colic • CholangitisCholangitis • HepatitisHepatitis • Fitz-Hugh-Curtis SyndromeFitz-Hugh-Curtis Syndrome (perihepatitis associated(perihepatitis associated c chlamydial infection of cervix)c chlamydial infection of cervix)
  • 99. Midepigastric:Midepigastric: • PUD non perforatedPUD non perforated • MIMI • EsophagitisEsophagitis • PEPE • PancreatitisPancreatitis • Herpes ZosterHerpes Zoster • Rectus sheat hematomaRectus sheat hematoma
  • 100. Acute abdomenAcute abdomen • RLQ or LLQ:RLQ or LLQ: • Ureteral calculiUreteral calculi • Regional enteritisRegional enteritis (Crohn’s Ds)(Crohn’s Ds) • InflammatoryInflammatory bowel Diseasebowel Disease (Regional enteritis,(Regional enteritis, UC)UC) • PIDPID • EndometriosisEndometriosis • ProstatitisProstatitis • MittelschmerzMittelschmerz • UTIUTI • Ruptured ovarianRuptured ovarian cystcyst • LUQ:LUQ: • LLL pneumoniaLLL pneumonia • Gastritis,Gastritis, • SplenomegalySplenomegaly
  • 101. Acute AbdomenAcute Abdomen Diffuse:Diffuse: • Abdominal wallAbdominal wall hematomahematoma • Spider biteSpider bite • Lead poisoningLead poisoning • Addisonian crisisAddisonian crisis • Sickle cell crisisSickle cell crisis • Diabetic ketoacidosisDiabetic ketoacidosis (DKA)(DKA) • DiabeticDiabetic gastropathygastropathy • Opiate withdrawalOpiate withdrawal
  • 102. GI emergenciesGI emergencies• AppendicitisAppendicitis • AscitesAscites • Biliary ColicBiliary Colic • Boerhaave SyndromeBoerhaave Syndrome • CholangitisCholangitis • Cholecystitis, AcuteCholecystitis, Acute • CirrhosisCirrhosis • Colitis, IschemicColitis, Ischemic • Colitis, UlcerativeColitis, Ulcerative • Crohn’s DiseaseCrohn’s Disease • DiverticulitisDiverticulitis • DiverticulosisDiverticulosis • GastritisGastritis • GastroenteritisGastroenteritis • Gastroesophageal RefluxGastroesophageal Reflux DiseaseDisease • HemorrhoidsHemorrhoids • Hepatic AbscessHepatic Abscess • Hepatic EncephalopathyHepatic Encephalopathy • Hepatitis, AlcoholicHepatitis, Alcoholic • Hepatitis, ViralHepatitis, Viral • Incarcerated HerniaIncarcerated Hernia • Intestinal ObstructionIntestinal Obstruction • Irritable Bowel SyndromeIrritable Bowel Syndrome • Mallory-Weiss SyndromeMallory-Weiss Syndrome • Meckel DiverticulumMeckel Diverticulum • Pancreatitis, AcutePancreatitis, Acute • Peptic Ulcer DiseasePeptic Ulcer Disease • Perforated peptic ulcerPerforated peptic ulcer • Variceal BleedingVariceal Bleeding
  • 103. Acute AbdomenAcute Abdomen Suprapubic:Suprapubic: • Ectopic pregnancyEctopic pregnancy • Ovarian torsionOvarian torsion • Tubo-ovarian abscessTubo-ovarian abscess • Incarcerated groin herniaIncarcerated groin hernia
  • 104. Which of the following is the MOSTWhich of the following is the MOST common cause of painful rectal bleeding?common cause of painful rectal bleeding? a. internal hemorrhoida. internal hemorrhoid b. external hemorrhoidb. external hemorrhoid c. diverticulitisc. diverticulitis d. anal fissured. anal fissure e. rectal foreign bodye. rectal foreign body
  • 106. Plain xray of abdomenPlain xray of abdomen
  • 107.
  • 108.
  • 109. ERCP Film showing a hypoecheoicERCP Film showing a hypoecheoic shadowshadow
  • 110. Plain xray chest showsPlain xray chest shows
  • 111. Plain xray abdomen in errect posturePlain xray abdomen in errect posture
  • 112. Plain xray abdomen in errect posturePlain xray abdomen in errect posture
  • 113.
  • 114.
  • 115.
  • 116.
  • 117.
  • 118. AnswerAnswer Intestinal ObstructionIntestinal Obstruction due to an Adhesivedue to an Adhesive BandBand
  • 119.
  • 121.
  • 123.
  • 124. AnswerAnswer » Intestinal Obstruction-Intestinal Obstruction- following obstructedfollowing obstructed Umbilical HerniaUmbilical Hernia
  • 125.
  • 126. AnswerAnswer • Intestinal ObstructionIntestinal Obstruction Incarcerated &Incarcerated & Strangulating &Strangulating & gangrenous umbilicalgangrenous umbilical Hernia at laparotomyHernia at laparotomy
  • 127.
  • 128. AnswerAnswer • Intestinal ObstructionIntestinal Obstruction Following an incisional HerniaFollowing an incisional Hernia with Incarceration andwith Incarceration and StrangulationStrangulation
  • 129.
  • 130. AnswerAnswer • Intestinal obstructionIntestinal obstruction alongwith gangrene ofalongwith gangrene of the small bowelthe small bowel following an incarceratedfollowing an incarcerated ventral herniaventral hernia
  • 131. A. Anal fissures result from a linear tear of theA. Anal fissures result from a linear tear of the anal canal beginning at or just below theanal canal beginning at or just below the dentate line and extending distally along thedentate line and extending distally along the anal canal.anal canal. B. Pt’s complain of sharp, cutting pain, mostB. Pt’s complain of sharp, cutting pain, most severe during and immediately after a bowelsevere during and immediately after a bowel movement.movement. C. Bleeding is scant and bright red.C. Bleeding is scant and bright red. D. Anal fissures are especially painful b/c of theD. Anal fissures are especially painful b/c of the rich supply of somatic sensory nerve fibersrich supply of somatic sensory nerve fibers located in the anodermlocated in the anoderm
  • 132. All of the following are true regarding acalculousAll of the following are true regarding acalculous cholecystitis EXCEPTcholecystitis EXCEPT A. It occurs in 5-10% of pt’s with acuteA. It occurs in 5-10% of pt’s with acute cholecystitischolecystitis B. Pt’s are frequently elderly and have aB. Pt’s are frequently elderly and have a history of DMhistory of DM C. It often occurs as a complication of anotherC. It often occurs as a complication of another processprocess D. Diagnosis is difficult due to the subtleD. Diagnosis is difficult due to the subtle clinical presentationclinical presentation E. Gallstones are absentE. Gallstones are absent Answer DAnswer D
  • 133. Answer DAnswer D • A calculus cholecystitis occurs in 5-10% of pt’sA calculus cholecystitis occurs in 5-10% of pt’s c cholecystitis.c cholecystitis. • Pts frequently are elderly and have h/o DMPts frequently are elderly and have h/o DM • There are 2 distinguishing features ofThere are 2 distinguishing features of acalculous cholecysititisacalculous cholecysititis – 1. it frequently occurs as a complication of1. it frequently occurs as a complication of another processanother process – 2. pts frequently are gravely ill on initial2. pts frequently are gravely ill on initial presentationpresentation
  • 134. Which of the following drugs is NOTWhich of the following drugs is NOT associated with acute pancreatitis?associated with acute pancreatitis? a. Heparina. Heparin b. Furosemideb. Furosemide c. Rifampinc. Rifampin d. Salicylatesd. Salicylates e. Warfarine. Warfarin
  • 135. Answer AAnswer A • Drugs and toxins are major causes ofDrugs and toxins are major causes of acute pancreatitis.acute pancreatitis. • Some of the meds assoc c the occurrenceSome of the meds assoc c the occurrence are OCP’s, glucocorticoids, rifampin,are OCP’s, glucocorticoids, rifampin, tetracycline, isoniazid, thiazide diuretics,tetracycline, isoniazid, thiazide diuretics, furosemide, salicylates, indomethacin,furosemide, salicylates, indomethacin, calcium, warfarin, and acetominophen.calcium, warfarin, and acetominophen. • Other etiologic factors contributingOther etiologic factors contributing include infection, collagen vascular ds’s,include infection, collagen vascular ds’s, metabolic disturbances, and traumametabolic disturbances, and trauma
  • 136. In working up a patient with acute abdominal pain,In working up a patient with acute abdominal pain, which of the following etiologies is LEAST likelywhich of the following etiologies is LEAST likely to represent an immediate life threat?to represent an immediate life threat? a. Myocardial infarctiona. Myocardial infarction b. Splenic ruptureb. Splenic rupture c. Abdominal Aortic Aneurysmc. Abdominal Aortic Aneurysm d. Perforated duodenal ulcerd. Perforated duodenal ulcer e. Ruptured ectopic pregnancye. Ruptured ectopic pregnancy
  • 137. Answer DAnswer D • When approaching a pt c acute abd pain, the clinicianWhen approaching a pt c acute abd pain, the clinician must consider conditions that can be an immediatemust consider conditions that can be an immediate threat to the patient’s life.threat to the patient’s life. • Splenic rupture, ruptured ectopic pregn, and AAA canSplenic rupture, ruptured ectopic pregn, and AAA can all be associated with massive bleeding and rapidall be associated with massive bleeding and rapid decline.decline. • Extra abdominal conditions that present with abd painExtra abdominal conditions that present with abd pain such as MI can also be life threatening.such as MI can also be life threatening. • Perforated duodenal ulcer are serious but almost neverPerforated duodenal ulcer are serious but almost never result in significant hemorrhage, and thus are notresult in significant hemorrhage, and thus are not usually an immediate threat to life.usually an immediate threat to life.
  • 138. Which of the following statements is TRUEWhich of the following statements is TRUE regarding acute abdominal pain?regarding acute abdominal pain? A. Peritonitis causes visceral type of pain and isA. Peritonitis causes visceral type of pain and is secondary to peritoneal inflammation from an irritantsecondary to peritoneal inflammation from an irritant B. Obstruction of a hollow viscus produces colicky,B. Obstruction of a hollow viscus produces colicky, diffuse visceral pain assoc with N/Vdiffuse visceral pain assoc with N/V C. Intra abdominal causes of pain include bacterialC. Intra abdominal causes of pain include bacterial peritonitis, bowel ischemia, and tub ovarian abscessperitonitis, bowel ischemia, and tub ovarian abscess D. referred pain from the abdomen may radiate to theD. referred pain from the abdomen may radiate to the back or groin, but not into the thoraxback or groin, but not into the thorax E. Metabolic disorders are rarely a significant sourceE. Metabolic disorders are rarely a significant source of acute abdominal painof acute abdominal pain
  • 139. Answer BAnswer B Three types of pain responses are possible with acute abd. painThree types of pain responses are possible with acute abd. pain  Peritonitis is a somatic pain and is usually sharper, morePeritonitis is a somatic pain and is usually sharper, more  constant, and more localized than visceral pain.constant, and more localized than visceral pain.  Obstruction of a hollow viscus is a common cause of visceralObstruction of a hollow viscus is a common cause of visceral  pain and is colicky, intermittent, and usually mid-line.pain and is colicky, intermittent, and usually mid-line.  Referred pain is often felt in the back, groin, or thighs. Pt’sReferred pain is often felt in the back, groin, or thighs. Pt’s  may also c/o pain in the supraclavicular region especially ifmay also c/o pain in the supraclavicular region especially if  the diaphragm is irritated by collections of blood or pus.the diaphragm is irritated by collections of blood or pus.  Abd pain can arise from intraabd, extraabd, metabolic, orAbd pain can arise from intraabd, extraabd, metabolic, or neurogenic origins.neurogenic origins.
  • 140. Intrabdominal origins of pain are divided intoIntrabdominal origins of pain are divided into 3 categories:3 categories: – Peritoneal inflame, obstruction of a hollowPeritoneal inflame, obstruction of a hollow viscus, and vascular etiologies.viscus, and vascular etiologies. – Extraabd sources can arise from the abd wall,Extraabd sources can arise from the abd wall, thorax, or pelvis (as in the case of tubo-ovarianthorax, or pelvis (as in the case of tubo-ovarian abscess)abscess) – Metabolic disorders such as DKA and Sickle cellMetabolic disorders such as DKA and Sickle cell crisis often present with diffuse abd paincrisis often present with diffuse abd pain
  • 141. All of the following are TRUE regarding theAll of the following are TRUE regarding the evaluation of a patient with acute abdominalevaluation of a patient with acute abdominal pain EXCEPTpain EXCEPT AA. The onset, location, and severity of pain are. The onset, location, and severity of pain are useful differentiating factorsuseful differentiating factors B. The most important physical examinationB. The most important physical examination modality is palpationmodality is palpation C. The WBC may be normal even in inflammatoryC. The WBC may be normal even in inflammatory conditions such as appendicitisconditions such as appendicitis D. Ultra-sonography is a valuable imaging toolD. Ultra-sonography is a valuable imaging tool increasingly availableincreasingly available E. Analgesic medications should be withheld until aE. Analgesic medications should be withheld until a surgeon evaluates the patient because they may obscuresurgeon evaluates the patient because they may obscure the diagnosis.the diagnosis.
  • 142. Answer EAnswer E • The evaluation of abd pain should begin with a detailed hx.The evaluation of abd pain should begin with a detailed hx. • The onset, severity, location, and character of pain and theThe onset, severity, location, and character of pain and the presence of associated symptoms guide work-up and tx.presence of associated symptoms guide work-up and tx. • Although a complete PE is necessary, palpation of the abd isAlthough a complete PE is necessary, palpation of the abd is the most important modality for dx.the most important modality for dx. • Lab tests are useful adjuncts, but the limitations of a CBCLab tests are useful adjuncts, but the limitations of a CBC must be recognized.must be recognized. • Helpful imaging modalities include standard XR, U/S,Helpful imaging modalities include standard XR, U/S, barium contrast studies, and CTbarium contrast studies, and CT • IV opiate analgesia is humane and may actually assist in dxIV opiate analgesia is humane and may actually assist in dx by facilitating PE in a pt who could otherwise not tolerate it.by facilitating PE in a pt who could otherwise not tolerate it.
  • 143. Which of the following is the most commonWhich of the following is the most common cause of upper GI bleeding?cause of upper GI bleeding? A. Esophageal varicesA. Esophageal varices B. Mallory-Weiss tearB. Mallory-Weiss tear C. PUDC. PUD D. Erosive gastritisD. Erosive gastritis E. Arteriovenous malformationsE. Arteriovenous malformations
  • 144. Answer CAnswer C • Upper GI bleeding is defined as bleeding that originates proximal to theUpper GI bleeding is defined as bleeding that originates proximal to the ligament of Treitz.ligament of Treitz. • PUD, including gastric, duodenal, and stomachal ulcers, is the MC cause ofPUD, including gastric, duodenal, and stomachal ulcers, is the MC cause of upper GI bleeding (60%)upper GI bleeding (60%) • The next MC are erosive gastritis, esophagitis, and duodenitis (15%)The next MC are erosive gastritis, esophagitis, and duodenitis (15%) • Gastric irritants such as ETOH, salicylates, and NSAIDS, predispose pts toGastric irritants such as ETOH, salicylates, and NSAIDS, predispose pts to upper GI bleeding.upper GI bleeding. • Varices (only 6%) from portal HTN in Ethers carry a high mortality rate.Varices (only 6%) from portal HTN in Ethers carry a high mortality rate. • Mallory-Weiss syndrome is due to a mucosal tear in the esophagus and isMallory-Weiss syndrome is due to a mucosal tear in the esophagus and is classically assoc c repeated bouts of retching.classically assoc c repeated bouts of retching. • AV malformations are an uncommon cause of upper GI bleedAV malformations are an uncommon cause of upper GI bleed
  • 145. A Pt presents with what appears to be a massiveA Pt presents with what appears to be a massive lower GI hemorrhage. Which one of the followinglower GI hemorrhage. Which one of the following is the LEAST likely etiology?is the LEAST likely etiology? A. DiverticulosisA. Diverticulosis B. AngiodysplasiaB. Angiodysplasia C. Gastric varicesC. Gastric varices D. Duodenal ulcerD. Duodenal ulcer E. HaemorrhoidsE. Haemorrhoids
  • 146. Answer EAnswer E • The MC cause of what initially appears to be lower GI bleeding isThe MC cause of what initially appears to be lower GI bleeding is actually bleeding from an upper GI source.actually bleeding from an upper GI source. • Brisk bleeding from either varices or PUD can be the cause ofBrisk bleeding from either varices or PUD can be the cause of apparent lower GI hemorrhage.apparent lower GI hemorrhage. • Diverticulosis and angiodysplasia are the MC causes of confirmedDiverticulosis and angiodysplasia are the MC causes of confirmed lower GI bleed.lower GI bleed. Both occur more commonly in elderly, are painless, and may beBoth occur more commonly in elderly, are painless, and may be massive.massive. • Although hemorrhoids are a common etiology of minor lower GIAlthough hemorrhoids are a common etiology of minor lower GI bleed, usually not significant hemorrhagebleed, usually not significant hemorrhage • Other less frequent sources of lower GI bleed include malignancies,Other less frequent sources of lower GI bleed include malignancies, IBD, polyps, infectious gastroenteritis, and Meckel’s diverticulum.IBD, polyps, infectious gastroenteritis, and Meckel’s diverticulum.
  • 147. Which of the following scenarios may representWhich of the following scenarios may represent acute appendicitis?acute appendicitis? A. a 4 y/o male with vomiting and lethargyA. a 4 y/o male with vomiting and lethargy B. a 75 y/o female with fever and abdominal painB. a 75 y/o female with fever and abdominal pain C. a 26 y/o female who is 32 weeks pregnant withC. a 26 y/o female who is 32 weeks pregnant with right upper quadrant painright upper quadrant pain D. a 45 y/o male with AIDS and who hasD. a 45 y/o male with AIDS and who has vomiting and diarrheavomiting and diarrhea E. All of the aboveE. All of the above
  • 148. Answer EAnswer E • Certain groups of pts have atypical presentations and are at risk forCertain groups of pts have atypical presentations and are at risk for delayed dx of acute appendicitis.delayed dx of acute appendicitis. • Children <6 y/o = 57% misdiagnosed; 90% perforation ratesChildren <6 y/o = 57% misdiagnosed; 90% perforation rates • Elderly pts, subtle sx, & high perforationsElderly pts, subtle sx, & high perforations • Pregnant pts pose difficulty b/c the gravid uterus changes thePregnant pts pose difficulty b/c the gravid uterus changes the position of the appendixposition of the appendix • An U/S can aid in distinguishing pelvic vs abd pathologyAn U/S can aid in distinguishing pelvic vs abd pathology • Immunocompromised are susceptible to delayed dx b/c of theirImmunocompromised are susceptible to delayed dx b/c of their frequent unrelated GI sx.frequent unrelated GI sx. • CT is helpful in differentiating surgical from nonsurgical conditionsCT is helpful in differentiating surgical from nonsurgical conditions
  • 149. What is the most common cause ofWhat is the most common cause of large bowel obstruction?large bowel obstruction? A. AdhesionsA. Adhesions B. Incarcerated herniaB. Incarcerated hernia C. DiverticulitisC. Diverticulitis D. NeoplasmD. Neoplasm E. Sigmoid volvulusE. Sigmoid volvulus
  • 150. Answer DAnswer D • It is important to distinguish between largeIt is important to distinguish between large and small bowel obstruction b/c tx differsand small bowel obstruction b/c tx differs • The MC cause of colonic obstruction is neoplasmThe MC cause of colonic obstruction is neoplasm • 22ndnd MC = diverticulitis, followed by sigmoidMC = diverticulitis, followed by sigmoid volvulusvolvulus • MC SBO = surgical adhesionsMC SBO = surgical adhesions • 22ndnd MC SBO = hernias and primary smallMC SBO = hernias and primary small bowel lesionsbowel lesions
  • 151. A 40 y/o female with known gallstones presents with colickyA 40 y/o female with known gallstones presents with colicky RUQ pain and vomiting. She has a h/o similar episodes thatRUQ pain and vomiting. She has a h/o similar episodes that usually resolve after 3-4 hrs. VS: BP 110/60, P 78, R 16, Tusually resolve after 3-4 hrs. VS: BP 110/60, P 78, R 16, T 98.4*F. PE: mildly tender RUQ without signs of peritonitis.98.4*F. PE: mildly tender RUQ without signs of peritonitis. Which of the following would be LEAST appropriate in her EDWhich of the following would be LEAST appropriate in her ED management?management? a. IV fluidsa. IV fluids b. Pain control c opiate analgesicsb. Pain control c opiate analgesics c. pain control c ketorolacc. pain control c ketorolac d. antiemetic administrationd. antiemetic administration e. immediate surgical consultatione. immediate surgical consultation
  • 152. Answer EAnswer E • Pts with uncomplicated symptomatic cholelithiasis do notPts with uncomplicated symptomatic cholelithiasis do not require immediate surgical intervention.require immediate surgical intervention. • ED intervention is geared toward pain relief and correction ofED intervention is geared toward pain relief and correction of volume deficitsvolume deficits • Pain control can be achieved with administration of opiates orPain control can be achieved with administration of opiates or ketorolacketorolac • Antiemetics and gastric decompression with an NGT may beAntiemetics and gastric decompression with an NGT may be necessary for tx of protracted vomiting.necessary for tx of protracted vomiting. • If the pt’s sx resolve within 4-6 hrs and she tolerates oralIf the pt’s sx resolve within 4-6 hrs and she tolerates oral fluids, D/C home with output f/u is appropriatefluids, D/C home with output f/u is appropriate
  • 153. Which of the following is the MOST commonWhich of the following is the MOST common presentation of gallstones?presentation of gallstones? a. acute pancreatitisa. acute pancreatitis b. acute cholecysitisb. acute cholecysitis c. biliary colicc. biliary colic d. ascending cholangitisd. ascending cholangitis e. gallbladder empyemae. gallbladder empyema
  • 154. Answer CAnswer C• Pt’s with gallstones present in a variety of ways, and biliaryPt’s with gallstones present in a variety of ways, and biliary colic (or symptomatic cholecystitis) is the MC.colic (or symptomatic cholecystitis) is the MC. • The pain is colicky in nature, occurs after meals, and typicallyThe pain is colicky in nature, occurs after meals, and typically lasts from 1-6 hrs.lasts from 1-6 hrs. • Pain lasting longer than 6 hrs that is accompanied by fever orPain lasting longer than 6 hrs that is accompanied by fever or leukocytosis suggests cholecystitis.leukocytosis suggests cholecystitis. • Biliary colic and acute cholecystitis are by far the MCBiliary colic and acute cholecystitis are by far the MC manifestations of gallstone dsmanifestations of gallstone ds • Complications of gallstones may be life threatening.Complications of gallstones may be life threatening. • Acute pancreatitis, ascending cholangitis, gallbladderAcute pancreatitis, ascending cholangitis, gallbladder empyema, and emphysematous cholecystitis all requireempyema, and emphysematous cholecystitis all require aggressive pt resuscitation and prompt surgical consultaggressive pt resuscitation and prompt surgical consult
  • 155. What is the MOST common cause of pancreatitisWhat is the MOST common cause of pancreatitis in an urban hospital setting?in an urban hospital setting? a. cholelithiasisa. cholelithiasis b. alcoholismb. alcoholism c. abdominal traumac. abdominal trauma d. penetrating peptic ulcerd. penetrating peptic ulcer e. salicylate poisoninge. salicylate poisoning
  • 156. Answer BAnswer B • Acute pancreatitis is a common cause of abdominalAcute pancreatitis is a common cause of abdominal pain.pain. • In the US cholelithiasis and alcoholism account forIn the US cholelithiasis and alcoholism account for 90%.90%. • ETOH related ds is more common in the urbanETOH related ds is more common in the urban setting, and typically affects males 35-45.setting, and typically affects males 35-45. • Biliary ds is more frequent in the community hospitalBiliary ds is more frequent in the community hospital setting and typically affects females >50.setting and typically affects females >50. • After biliary and ETOH = drugs (1/2 of remaining cases)After biliary and ETOH = drugs (1/2 of remaining cases)
  • 157. A 55 y/o female presents to the ED with a feverA 55 y/o female presents to the ED with a fever 4 days after undergoing a laparscopic4 days after undergoing a laparscopic cholecystectomy. What is the MOST likelycholecystectomy. What is the MOST likely cause of the fever?cause of the fever? a. Pneumoniaa. Pneumonia b. Thrombophlebitisb. Thrombophlebitis c. Urinary Tract infectionc. Urinary Tract infection d. Wound infectiond. Wound infection e. Deep venous thrombosise. Deep venous thrombosis
  • 158. Answer CAnswer C• Laparoscopic procedures and early postsurgicalLaparoscopic procedures and early postsurgical discharge are becoming increasingly commondischarge are becoming increasingly common cost-effective alternatives to laparotomy.cost-effective alternatives to laparotomy. • As a result, more pts are presenting to the EDAs a result, more pts are presenting to the ED with postoperative fever.with postoperative fever. • Fever postopFever postop – <24 h = atelectasis or necrotizing strept<24 h = atelectasis or necrotizing strept infectionsinfections – 24-72 h = respiratory (pneumonia), IV catheter24-72 h = respiratory (pneumonia), IV catheter complications (thrombophlebitis)complications (thrombophlebitis) – 3-5 d = UTI’s (MC in female, and pts c cath’s)3-5 d = UTI’s (MC in female, and pts c cath’s) – 7-10 d = wound infections7-10 d = wound infections – DVT’s can result in fever @ any time, pero usually >5dDVT’s can result in fever @ any time, pero usually >5d
  • 159. A pt with suspected cholelithiasis presents to the ED.A pt with suspected cholelithiasis presents to the ED. What is the initial imaging study of choice?What is the initial imaging study of choice? a. abdominal plain filma. abdominal plain film b. abdominal ultrasoundb. abdominal ultrasound c. abdominal CTc. abdominal CT d. Radionuclide scan (HIDA)d. Radionuclide scan (HIDA) e. Barium contrast radiographye. Barium contrast radiography
  • 160. Answer BAnswer B • U/S has emerged as a valuable tool for certain conditions in the ED.U/S has emerged as a valuable tool for certain conditions in the ED. • It is the initial study of choice for eval of pts with RUQ pain,It is the initial study of choice for eval of pts with RUQ pain, and can accurately detect cholelithiasis.and can accurately detect cholelithiasis. • Plain film is a poor imaging choice to detect gallstones (onlyPlain film is a poor imaging choice to detect gallstones (only 15%), but is useful in evaluating obstruction or suspected perforation.15%), but is useful in evaluating obstruction or suspected perforation. • CT is the diagnostic tool of choice for many abdominal conditionsCT is the diagnostic tool of choice for many abdominal conditions including pancreatitis, some trauma, and selected AAA, but is moreincluding pancreatitis, some trauma, and selected AAA, but is more costly and invasive than U/S for evaluating gallstones.costly and invasive than U/S for evaluating gallstones. • HIDA scan is a useful adjunct if U/S results are inconclusive orHIDA scan is a useful adjunct if U/S results are inconclusive or acalculous cholecysititis is suspected.acalculous cholecysititis is suspected. • Barium studies are useful for imaging in some GI conditions, especiallyBarium studies are useful for imaging in some GI conditions, especially suspected intussusception, but not for eval of GBsuspected intussusception, but not for eval of GB
  • 161. Which of the following diagnostic study is mostWhich of the following diagnostic study is most useful in the evaluation of a patient in the acuteuseful in the evaluation of a patient in the acute stages of diverticulitis?stages of diverticulitis? a. CT scana. CT scan b. barium enemab. barium enema c. ultrasound of the abdomenc. ultrasound of the abdomen d. colonoscopyd. colonoscopy e. sigmoidoscopye. sigmoidoscopy
  • 162. Answer AAnswer A • Both endoscopy and barium enema areBoth endoscopy and barium enema are contraindicated in pts during acute stages ofcontraindicated in pts during acute stages of diverticulitis b/c of risk of perforation.diverticulitis b/c of risk of perforation.
  • 163. A 28 y/o man has complaints of intermittent, colicky,A 28 y/o man has complaints of intermittent, colicky, periumbilical, and lower-quadrant pain for 24 hours. Theperiumbilical, and lower-quadrant pain for 24 hours. The patient admits to nausea and decreased appetite. He is afebrile.patient admits to nausea and decreased appetite. He is afebrile. Which of the following is the most likely diagnosis?Which of the following is the most likely diagnosis? a. acute appendicitisa. acute appendicitis b. acute pancreatitisb. acute pancreatitis c. pyelonephritisc. pyelonephritis d. gastroenteritisd. gastroenteritis e. peptic ulcer diseasee. peptic ulcer disease
  • 164. Answer DAnswer D • The pain pattern is most consistent with a dx ofThe pain pattern is most consistent with a dx of gastroenteritis.gastroenteritis. • Acute appendicitis typically causes periumbilical pain thatAcute appendicitis typically causes periumbilical pain that migrates to the RLQmigrates to the RLQ • Acute pancreatitis radiates to the back or shoulderAcute pancreatitis radiates to the back or shoulder • Pyelonephritis is “loin to groin”Pyelonephritis is “loin to groin” • PUD is typically located in the epigastrumPUD is typically located in the epigastrum
  • 165. A 7 y/o boy presents with c/o flank pain, fever,A 7 y/o boy presents with c/o flank pain, fever, frequency, dysuria, and hematuria for 1 day. The UAfrequency, dysuria, and hematuria for 1 day. The UA shows >10 WBC’s per high-powered field, RBC’s, andshows >10 WBC’s per high-powered field, RBC’s, and WBC casts. Of the following, the most likelyWBC casts. Of the following, the most likely diagnosis is:diagnosis is: a. pyelonephritisa. pyelonephritis b. acute cystitisb. acute cystitis c. urethritisc. urethritis d. renal calculid. renal calculi e. urinary incontinencee. urinary incontinence
  • 166. Answer AAnswer A • Pyelonephritis is an infection of the renal parenchyma,Pyelonephritis is an infection of the renal parenchyma, accompanied by systemic sx such as fever, N/V and assocaccompanied by systemic sx such as fever, N/V and assoc with WBC casts in the urine.with WBC casts in the urine. • In pediatric male pt, its occurrence would warrantIn pediatric male pt, its occurrence would warrant additional eval to r/o anatomic abnormalities in theadditional eval to r/o anatomic abnormalities in the urinary tracturinary tract
  • 167. The most common cause of acute renal failure is :The most common cause of acute renal failure is : a. prerenal azotemiaa. prerenal azotemia b. acute parenchymal renal failureb. acute parenchymal renal failure c. exogenous nephrotoxinsc. exogenous nephrotoxins d. obstructive uropathyd. obstructive uropathy e. rhabdomyolysise. rhabdomyolysis
  • 168. Answer AAnswer A • Prerenal azotemia results from renal hypoperfusionPrerenal azotemia results from renal hypoperfusion and can be reversed upon restoration of blood flow.and can be reversed upon restoration of blood flow. • It is not associated with structural damage to theIt is not associated with structural damage to the kidney and is the most common cause of acute renalkidney and is the most common cause of acute renal failure.failure.
  • 169. y/o woman with amenorrhea is seen for vaginaly/o woman with amenorrhea is seen for vaginal bleeding and abdominal pain. An ectopic pregnancy isbleeding and abdominal pain. An ectopic pregnancy is suspected. Which of the following would support thesuspected. Which of the following would support the suspicion?suspicion? a. enlarged boggy uterusa. enlarged boggy uterus b. ruptured fetal membranesb. ruptured fetal membranes c. adnexal massc. adnexal mass d. weak fetal heart beatd. weak fetal heart beat e. painless profuse bleedinge. painless profuse bleeding Answer CAnswer C Classic features of an ectopic pregnancy are abdominal pain,Classic features of an ectopic pregnancy are abdominal pain, bleeding, and adnexal mass in a pregnant woman.bleeding, and adnexal mass in a pregnant woman.